Как определить скорость по тормозному пути: Расчет тормозного пути

Содержание

Вот как можно рассчитать тормозной путь: Формула

Как рассчитать расстояние тормозного пути автомобиля.

 

Как быстро автомобиль ускоряется, наверное, знает большинство автовладельцев. Даже если вы не замеряли динамику разгона своей машины, вы наверняка смотрели заводские технические характеристики вашего авто, где обычно автопроизводитель указывает минимально возможное время разгона с 0-100 км/час. Но теперь вопрос: сколько времени нужно, чтобы остановить вашу машину? Вы знаете это? Уверены, что нет. Но, оказывается, рассчитать расстояние тормозного пути можно достаточно легко с помощью простой формулы. Мы расскажем вам, как это делается. 

 

Нет такой вещи во Вселенной или материи, которая может мгновенно остановиться. Также и любой автомобиль, когда вы нажимаете педаль тормоза, не сразу может остановиться. Дело в том, что для того чтобы автомобиль или любой объект в нашем мире остановился, необходимо, чтобы он потерял энергию, которая его движет.

В результате у любого автомобиля есть тормозной путь, который он проезжает с момента нажатия педали тормоза до момента полной остановки. Это и есть тормозное расстояние машины.

 

Но на самом деле тормозной путь любого авто зависит не только от его характеристик и тормозной системы, но и от реакции водителя при нажатии педали тормоза. Ведь для того чтобы принять решение о необходимости торможения и нажать педаль тормоза, требуется время, которое хоть и минимально, но достаточно, чтобы машина успела проехать немаленький путь. Особенно это важно при большой скорости движения, где за какие-то доли секунды автомобиль проезжает приличное расстояние. Итак, в итоге, чтобы рассчитать реальную длину тормозного пути, нужно учитывать не только время и расстояние, пройденное автомобилем с момента нажатия водителем педали тормоза до момента остановки машины, но и время, необходимое для принятия решения о торможении. Дело в том, что при принятии решения о торможении мы тратим драгоценные секунды.

Вот пример:

 

  • Время отклика: Прежде чем водитель нажмет педаль тормоза, он должен оценить дорожную ситуацию и определить, необходимо ли торможение. Также нужно понять, какое необходимо торможение – полная остановка автомобиля или простое снижение скорости. Обычно, согласно многочисленным исследованиям, большинству водителей для этого требуется около 0,1 секунды. 
  • Время, необходимое для нажатия педали тормоза: После того, как водитель понял, что должен тормозить, необходимо еще примерно 0,8 секунды, для того чтобы переместить ногу с педали газа на педаль тормоза и нажать ее. 

 

Кроме того, даже при нажатии педали тормоза есть еще небольшая потеря времени, связанная с тем, что при нажатии педали тормоза автомобиль, как правило, не начинает резко тормозить. А для того чтобы машина реально начала резко снижать скорость, надо усилить давление на педаль тормоза (пороговое время, необходимое для требуемого тормозного давления в тормозной системе).

Также у всех автомобилей разное время отклика на нажатую педаль тормоза. Здесь все, конечно, зависит от конструкции тормозной системы и наличия различной электроники, контролирующей тормоза автомобиля.

 

Смотрите также: Полный привод оказался лучше при торможении, чем привод на два колеса: Видео

 

Вы не поверите, но для того чтобы машина реально начала тормозить после нажатия педали тормоза, необходима еще почти 1 секунда времени. Вы представляете, как это много при движении на большой скорости? За эту лишнюю секунду вы можете проехать очень большой путь. 

 

Что такое формула тормозного пути?

В общем, торможение автомобиля делится на два вида. Например, есть нормальное торможение, а есть экстренное, когда вам нужно резко остановить машину, чтобы избежать аварии.

 

При торможении в повседневной жизни, допустим, если вы хотите остановить автомобиль на светофоре, вы обычно нажимаете педаль тормоза намного плавнее и мягче, чем при необходимости полностью остановить автомобиль на парковке во дворе. В этом случае вы не применяете в машине максимальное тормозное усилие. При таком плавном и мягком торможении, как правило, тормозной путь (тормозное расстояние) увеличивается. Примерное расстояние тормозного пути при 

нормальном торможении можно рассчитать по следующей простой формуле:

 

(Скорость в км/ч : 10) x (скорость в км/ч : 10) = тормозной путь в метрах

 

При экстренном торможении педаль тормоза, как правило, нажата целиком и с полной силой. Из-за более высокой силы торможения обычно тормозной путь машины сокращается примерно в 2 раза. Поэтому длину тормозного пути можно также вычислить по следующей формуле:

 

(Скорость в км/ч : 10) x (скорость в км/ч : 10) / 2 = тормозной путь в метрах

 

Внимание: Вычисляемый по этим формулам тормозной путь является лишь приблизительным значением и подсказкой для водителей. На самом деле в реальности тормозной путь может быть как меньше, так и больше. Ведь расстояние тормозного пути зависит от навыков и опыта вождения водителя, от технической исправности автомобиля, его конструкции, марки, модели, состояния дорог, состояния протектора резины и многих других факторов, которые напрямую влияют на длину тормозного пути. Но благодаря этим формулам вы примерно сможете высчитать среднюю длину тормозного пути машины при определенной скорости движения. Это позволит вам скорректировать ваш стиль управления автомобилем, а также станет хорошим пособием для водителей-новичков. 

 

Как рассчитать полное время остановки и итоговый тормозной путь?

 

Как мы уже сказали, чтобы рассчитать весь тормозной путь, нужно учитывать потерю времени при принятии водителем решения о торможении (то есть время реакции водителя). Для этого нужно использовать другую формулу, которая обеспечивает более точный приблизительный расчет тормозного расстояния, которое проедет автомобиль в момент принятия решения о необходимости остановки. Вот эта формула:

 

(Скорость в км/ч : 10) x 3 = путь реакции в метрах

 

В итоге, сделав вычисление по вышеуказанным формулам, вы можете вычислить приблизительный итоговый тормозной путь вашего автомобиля при любой скорости движения.

Вот пример. Если вы управляете своим автомобилем со скоростью 50 км/ч, то с помощью приведенных формул вычислите следующие значения:

 

  • Тормозной путь при принятии решения о торможении на этой скорости (реакция на дорожную ситуацию + принятие решения о торможении + время, необходимое для перемещения ноги с педали газа на педаль тормоза, а также время отклика тормозной системы на нажатую педаль тормоза) составит где-то (50/10) х 3 = 15 метров. То есть пока вы будете принимать решение о торможении при скорости в 50 км/ч, ваша машина проедет 15 метров. 
  • Тормозной путь при нормальном торможении (с момента нажатия педали тормоза до момента остановки машины) составит около (50/10) х (50/10) = 25 метров. 
  • При экстренном торможении тормозной путь, как мы уже отметили, сокращается примерно в два раза. Соответственно, расчет тормозного расстояния автомобиля, который движется со скоростью 50 км/ч, будет выглядеть следующим образом: (50/10) x (50/10) / 2 = 12,5 метров.
  • В результате теперь мы можем вычислить реальный итоговый тормозной путь автомобиля. Так, при нормальном (не резком, а обычном) торможении итоговый тормозной путь составит около 40 метров. При экстренном торможении – не менее 28 метров. 

 

Примечание: Обратите внимание, что если скорость автомобиля будет выше всего в два раза, его итоговый тормозной путь увеличится в четыре раза!!!

 

Смотрите также: Основные принципы работы тормозного механизма автомобиля [Принцип работы и элементы тормозной системы]

 

То есть мнение о том, что при увеличении скорости автомобиля в два раза тормозной путь увеличивается только в два раза, – это чистый воды миф среди многих автолюбителей. Так что имейте это в виду, когда садитесь за руль. Самое удивительное, что об этом не знают даже многие опытные водители. 

 

Пример расчета тормозных и остановочных расстояний

Скорость, в км / ч

Путь, пройденный автомобилем

во время реакции водителя, в метрах

Тормозное расстояние, в метрах

(с момента нажатия педали тормоза

до полной остановки машины)

Итоговый тормозной путь, в метрах

25

7,5

6,25

13,75

50

15

25

40

100

30

100

130

150

45

225

265

200

60

400

460

 

Какие факторы влияют на торможение и тормозной путь?

 

Решающим значением для длины тормозного пути, конечно же, является скорость автомобиля, с которой он движется по дороге. Также на тормозной путь влияет качество установленной на машину тормозной системы. В том числе важную роль, несомненно, играет и состояние дороги (снег, лед, качество асфальта/бетона, трещины в дорожном покрытии, листья, лужи и т. п.). И само собой, не стоит забывать о состоянии шин автомобиля. Ведь в определенных случаях изношенная резина сильно увеличит тормозной путь автомобиля, так как не сможет передавать нормальную тормозную способность дорожному покрытию в отличие от новых шин, имеющих нормальное сцепление с дорогой. 

Также ясно, что на мокрой поверхности тормозное расстояние машины больше, чем на сухом асфальте. 

 

Не стоит забывать и об уровне подготовки водителя. Особенно важна, как мы узнали, для итогового тормозного пути скорость реакции водителя на дорожную ситуацию, требующую остановки автомобиля. Но скорость реакции за рулем зависит не только от опыта вождения. Например, знаете ли вы, что когда вы садитесь за руль в сонном состоянии (не выспались, устали или долго находились за рулем), то скорость реакции может замедлиться почти в два раза по сравнению со скоростью реакции хорошо отдохнувшего водителя.  

В целом же на скорость принятия решения за рулем (скорость реакции) влияет много факторов: возраст водителя, алкогольное или похмельное состояние, употребление определенных медикаментов и в целом состояние здоровья. Так, при многих хронических заболеваниях скорость реакции многих водителей существенно снижается. Следовательно, все эти факторы серьезно влияют на тормозной путь автомобиля. 

 

Смотрите также: Тормозной путь автомобиля: Все что нужно знать

 

То же самое касается и отвлечения внимания из-за смартфонов, которыми так любят пользоваться за рулем многие водители, несмотря на строгий запрет согласно нашему действующему законодательству.

 

Как мы уже сказали, на тормозной путь также влияет время отклика тормозной системы автомобиля на нажатую педаль тормоза. Особенно это касается старых автомобилей. Современные же, как правило, оснащены уже новым поколением тормозов, которые мгновенно активируются за счет максимального тормозного давления, как только вы резко ударите ногой по педали тормоза (например, при экстренном торможении). Эта технология позволила существенно сократить итоговый тормозной путь современных машин. 

 

Как повысить безопасность при управлении автомобилем?

 

Не зря основное правило вождения гласит о том, что водитель должен держать на дороге достаточную дистанцию до других автомобилей, чтобы оставалось пространство для экстренного торможения и для того, чтобы не спровоцировать ДТП. Но, с другой стороны, вы не должны держать дистанцию между автомобилями слишком большой. Помните, что все должно быть в меру. Вот некоторые правила вождения от экспертов:

 

  • В городском движении: Держите расстояние до других автомобилей около 15 метров. 
  • На автомагистралях, шоссе и проселочных дорогах: При скорости движения около 100 км/ч держите дистанцию примерно 50 метров. При плохой видимости или на скользкой дороге дистанция до других машин должна быть увеличена в два раза. Например, при скорости в 100 км/ч на скользкой дороге держите расстояние до впереди идущей машины минимум в 100 метров.  

Калькулятор остановочного пути автомобиля • Механика • Онлайн-конвертеры единиц измерения

Калькулятор определяет остановочный путь автомобиля с момента обнаружения водителем опасности до момента полной остановки автомобиля, а также другие параметры, связанные с этим событием, в частности, время восприятия водителем сигнала о необходимости торможения, время реакции водителя, а также расстояние, которое прошел автомобиль во время этих событий. Калькулятор также определяет начальную скорость (скорость до начала торможения) по известной длине торможения (длины тормозного пути) с учетом дорожных условий. Как и все остальные калькуляторы, этот калькулятор не следует использовать в судебных процессах и при необходимости получения высокой точности.

Пример 1: Рассчитать расстояние, необходимое для остановки автомобиля, движущегося со скоростью 90 км/ч по мокрой горизонтальной дороге с асфальтобетонным покрытием (коэффициент трения μ = 0,4) если время восприятия водителя 0,5 с и время реакции водителя 0,7 с.

Пример 2: Рассчитать начальную скорость автомобиля, движущегося по дороге с мокрым асфальтобетонным покрытием (μ = 0.4), если длина тормозного пути равна 100 м. Автомобиль движется на спуске с уклоном 10%.

Калькулятор остановочного пути

Входные данные

Начальная скорость

v0м/скм/чфут/смиля/ч

Время восприятия опасности водителем

thpс

Время реакции водителя

thrс

Уклон

σградус%

Движение вверх Движение вниз

Состояние дороги

—Сухой асфальтМокрый асфальтПокрытый снегом асфальтПокрытый льдом асфальт

или Коэффициент трения

μ

Тип привода тормозов

—ПневматическийГидравлический

или Время срабатывания тормозной системы

tbrlс

Выходные данные

Угол крутизны уклона θ= °

Замедление a= м/с²

Время торможения tbr= с

Расстояние, которое проедет автомобиль во время восприятия водителем опасности Shp= м

Расстояние, которое проедет автомобиль во время реакции водителя на опасность Shr= м

Расстояние, которое проедет автомобиль за время задержки срабатывания тормоза Sbrl= м

Тормозной путь Sbr= м

Остановочный путь Sstop= м

Критический угол наклона для заданного коэффициента трения θcrit= °

Критический уклон для заданного коэффициента трения σcrit= %

Определения и формулы

Остановочный путь

Остановочный путь — это расстояние, которое проходит автомобиль с момента, когда водитель видит опасность, оценивает ее, принимает решение остановиться и нажимает на педаль тормоза и до момента полной остановки автомобиля. Это расстояние является суммой нескольких расстояний, которые проходит автомобиль в то время, как водитель принимает решение, срабатывают механизмы тормозной системы и происходит замедление движения до полной остановки.

где shr — расстояние, которое проедет автомобиль во время восприятия и оценки водителем ситуации, shr — расстояние, которое проедет автомобиль во время во время реакции водителя на ситуацию, sbrl — расстояние, которое проедет автомобиль во время задержки срабатывания тормозов, и sbr — тормозной путь.

Расстояние, которое пройдет автомобиль во время восприятия и оценки водителем ситуации

Расстояние человеческого восприятия ситуации — это расстояние, которое пройдет автомобиль в то время, пока водитель оценивает опасность и принимает решение уменьшить скорость и остановиться. Оно определяется по формуле

где shp расстояние человеческого восприятия в метрах, v скорость автомобиля в км/ч, thp — время человеческого восприятия в секундах и 1000/3600 — коэффициент преобразования километров в час в метры в секунду (1 километр равен 1000 метров и 1 час равен 3600 секундам).

Расстояние, которое пройдет автомобиль во время реакции водителя

Расстояние реакции водителя — это расстояние, которое пройдет автомобиль пока водитель выполняет решение остановить автомобиль после оценки опасности и принятия решения об остановке. Оно определяется по формуле

где shp — расстояние реакции водителя с метрах, v — скорость автомобиля в км/ч и thr — время реакции водителя в секундах.

Расстояние, которое пройдет автомобиль во время срабатывания тормозной системы

Расстояние, которое пройдет автомобиль во время срабатывания тормозной системы, зависит от типа тормозной системы, установленной на автомобиле. Почти на всех легковых автомобилях и малотоннажных грузовых автомобилях используются гидравлическая тормозная система. На большинстве большегрузных автомобилей используются тормоза с пневматическим приводом. Задержка срабатывания пневматических тормозов приблизительно равна 0,4 с, а гидравлических (жидкость несжимаема!) 0,1–0,2 с. Общая задержка срабатывания тормозной системы измеряется как время от момента нажатия на педаль тормоза, в течение которого замедление становится устойчивым. Оно состоит из задержки срабатывания тормозной системы и времени установления постоянной величины замедления движения. В тормозной системе с пневматическим приводом воздуху необходимо время, чтобы пройти по тормозным магистралям. С другой стороны, в гидравлическом приводе задержек практически не наблюдается, и он работает в два—пять раз быстрее, чем пневматический.

Расстояние, которое пройдет автомобиль во время срабатывания тормозной системы, определяется по формуле

где sbrl — расстояние в метрах, которое пройдет автомобиль во время срабатывания тормозной системы, v — скорость движения автомобиля в км/ч, tbrl — время срабатывания тормозной системы в секундах.

Замедление

Для упрощения расчетов предположим, что автомобиль движется с постоянным ускорением или замедлением, которое определяется по известной из курса элементарной физики формуле равноускоренного или равнозамедленного движения

где a — ускорение, v — начальная скорость, v0 — конечная скорость и t — время.

Тормозной путь автомобиля

Тормозной путь автомобиля — это расстояние, которое проходит автомобиль с момента полного нажатия на педаль тормоза до момента полной остановки. Это расстояние зависит от скорости автомобиля перед началом торможения и от коэффициента трения между шинами и дорожным покрытием. В этом калькуляторе мы не учитываем другие факторы, влияющие на тормозной путь, например, сопротивление качению шин или лобовое сопротивление воздуха

В результатах исследования1, в котором коэффициент трения определялся путем измерения замедления, определено, что антиблокировочная тормозная система (АБС) влияла на коэффициент трения таким образом: он увеличивается с увеличением скорости при использовании АБС и уменьшается, если АБС не используется. В этом исследовании также подтверждается, что на коэффициент трения между шинами и дорожным покрытием влияет температура и интенсивность дождя.

Вывод зависимости тормозного пути от скорости и трения с использованием второго закона Ньютона

Коэффициент трения определяется как отношения силы трения к силе нормального давления, прижимающей тело к опоре:

или

где Ffr — сила трения, μ коэффициент трения и Fnorm — сила реакции опоры.

Действующая на тело нормальная сила реакции опоры определяется как составляющая силы реакции, перпендикулярная к поверхности опоры тела. В простейшем случае, когда тело находится на плоской горизонтальной поверхности, нормальная сила равна весу этого тела:

где m — масса тела и g — ускорение свободного падения. Эта формула выведена из второго закона Ньютона:

В более сложном случае, если тело расположено на наклонной плоскости, нормальная сила рассчитывается как

где θ — угол наклона между плоскостью поверхности и горизонтальной плоскостью. В этом случае нормальная сила меньше веса тела. Случай наклонной поверхности мы рассмотрим чуть позже.

В случае же горизонтальной поверхности, если коэффициент трения между телом и поверхностью равен μ, то сила трения равна

В соответствии со вторым законом Ньютона, эта сила трения, приложенная к движущемуся телу (автомобилю) приводит к возникновению пропорционального ей замедления:

или

Теперь, в соответствии с уравнением ускоренного (замедленного) движения имеем

Из курса элементарной физики известно, что при равнозамедленном движении с постоянным замедлением, если конечная скорость равна нулю, то тормозной путь определяется уравнением

Это уравнение можно переписать в более удобной форме с использованием преобразования скорости в км/час в м/с:

Подставляя в это уравнение a = μg, получаем формулу тормозного пути:

где скорость v задается в км/час, а ускорение силы тяжести g в м/с².

Решая это уравнение относительно v, получаем:

Аналогичную формулу для определения тормозного пути можно получить с помощью энергетического метода.

Вывод зависимости тормозного пути от скорости и трения с помощью энергетического метода

Теоретическое значение тормозного пути можно найти, если определить работу по рассеиванию кинетической энергии автомобиля. Если автомобиль, движущийся со скоростью v, замедляет движение до полной остановки, работа тормозной системы Wb, требуемая для полного рассеяния кинетической энергии автомобиля Ek, равна этой энергии:

Кинетическая энергия движущегося автомобиля Ek определяется формулой

где m — масса автомобиля и v — скорость движения автомобиля перед началом торможения.

Работа Wb, выполненная тормозной системой, определяется как

где m — масса автомобиля, μ — коэффициент трения между шинами и дорожным покрытием, g — ускорение силы тяжести и sbr — тормозной путь, то есть расстояние, которое прошел автомобиль от начала торможения до полной остановки.

Теперь, с учетом того, что Ek = Wb, имеем:

или

Скорость автомобиля до начала торможения является наиболее важным фактором, влияющим на величину остановочного пути. Другими, менее важными, факторами, влияющими на остановочный путь, являются время оценки водителем ситуации, время реакции водителя, скорость работы тормозной системы автомобиля и состояние дороги.

Время торможения

Из курса элементарной физики известно, что средняя скорость при равноускоренном движении равна полусумме начальной и конечной скорости:

С учетом, что конечная скорость равна нулю, время торможения определяется в калькуляторе как

Движение вверх и вниз по уклону

Силы, действующие на автомобиль на уклоне: Fg — сила тяжести (вес автомобиля), Fgd — скатывающая вниз составляющая веса автомобиля, Ffr — сила трения, действующая параллельно поверхности дорожного полотна с уклоном, Fgn — нормальная составляющая веса автомобиля, направленная перпендикулярно поверхности дороги, и Fnr — сила реакции опоры, равная нормальной составляющей веса автомобиля.

Когда водитель нажимает на педаль тормоза, замедляющий движение автомобиль может быть представлен в виде тела на поверхности с углом наклона θ (см. рисунок выше). Для простоты мы будем рассматривать только две силы, действующие на автомобиль, находящийся на уклоне. Это вес автомобиля и сила трения. Автомобиль, движущийся с начальной скоростью, замедляет движение, если сила трения, действующая параллельно дорожному полотну, больше, чем скатывающая сила, являющаяся составляющей силы тяжести, которая также параллельна дорожному полотну. Если начальная скорость автомобиля равна нулю, он в этой ситуации остается на месте при условии, что угол уклона меньше критического (об этом — ниже).

В то время, как сила тяжести Fg стремится скатывать автомобиль вниз, сила трения Ffr сопротивляется этому движению. Чтобы автомобиль мог в этой ситуации остановиться, сила трения должна превышать скатывающую составляющую силы тяжести Fgd.

В то же время, если сила трения превышает скатывающую составляющую силы тяжести, автомобиль будет двигаться вниз с постоянным ускорением и его тормозная система будет неспособна его остановить. Это может произойти, если угол наклона (уклон) дорожного полотна слишком велик или коэффициент трения слишком мал (вспомним как ведет себя автомобиль с обычными шинами на уклоне, если он покрыт коркой льда!).

По определению коэффициента трения, можно записать уравнение для силы трения:

или

Скатывающая составляющая силы тяжести:

Результирующая сила Ftotal, действующая на автомобиль на уклоне:

или

Как мы уже отмечали, сила Ftotal должна быть направлена вверх, иначе автомобиль при движении вниз остановить невозможно. В соответствии со вторым законом Ньютона, ускорение (точнее, замедление) автомобиля, движущегося под действием силы Ftotal, определяется как

Подставляя ускорение в выведенную выше формулу тормозного пути, получаем:

Решая это уравнение для vpre-braking, получим:

Отметим еще раз, что в этих формулах g задается в м/с, v в км/ч и s в метрах. В нашем калькуляторе используются две последние формулы.

Припаркованные и движущиеся по ул. Дивисадеро в Сан-Франциско (Калифорния) автомобили. Уклон дорожного полотна в этом месте равен 31% или 17°.

Уклон

Величина уклона дороги (показателя крутизны склона) равна тангенсу угла плоскости дорожного покрытия к горизонтали. Он рассчитывается как отношение перпендикуляра, опущенного из точки на поверхность (превышения местности) к длине горизонтальной поверхности от начала склона до перпендикуляра (горизонтальному расстоянию). По определению уклона считается, что при движении вверх уклон является положительным, а при движении вниз уклон является отрицательным, когда превышение в действительности является понижением дороги. Уклон дороги σ выражают как угол наклона к горизонтали в градусах или как отношение в процентах. Например, подъёму 15 метров на 100 метров перемещения по горизонтали соответствует уклон, равный 0,15 или 15%. В этом калькуляторе мы используем уклон в процентах, определяемый по формуле

где Δh — превышение местности и d — проекция уклона на горизонталь (см. рисунок выше). Если известен уклон, то угол наклона можно определить по формуле

Критический угол

При увеличении угла наклона дорожного полотна выше определенного значения, называемого критическим углом, движущийся вниз автомобиль затормозить невозможно, так как действующая на него сила трения становится меньше скатывающей силы. Этот критический угол находится из условия

или

или

Из этой формулы можно найти критический угол для данного коэффициента трения, при котором автомобиль не сможет затормозить:

Уклон, выраженный в процентах, определяется по известному углу наклона таким образом:

Пример

В этом примере мы покажем, как использовать формулу для определения тормозного пути. Пусть автомобиль движется с начальной скоростью vpre-braking = 90 км/ч вниз по уклону σ = 5% по мокрому асфальту (коэффициент трения μ = 0,4). Нужно определить тормозной путь. Для расчетов используем выведенные выше формулы.

Особые случаи

Нажмите на соответствующую ссылку, чтобы посмотреть как работает калькулятор в особых режимах:

Литература

  1. Hartman, J 2014, Effects of velocity, temperature And rainfall on the friction coefficient of pneumatic tyres And bitumen roads, Doctor of Philosophy (PhD), Aerospace, Mechanical And Manufacturing Engineering, RMIT University PDF 48 MB
  2. Wikibooks. Fundamentals of Transportation

Формула нахождения тормозного пути


Тормозной путь — калькулятор, формула и расчет онлайн

Калькулятор тормозного пути позволит оценить тормозной путь автомобиля, движущегося с заданной скоростью. Для использования укажите тип дорожного покрытия, на котором тормозит автомобиль и скорость, при которой начинается торможение. Калькулятор рассчитает сколько метров пройдет автомобиль при торможении.

Калькулятор тормозного пути

Формула тормозного пути

Формула для нахождения тормозного пути

Формула для нахождения тормозного пути применяется в подразделениях ГИБДД. Именно она используется в нашем калькуляторе. В этой формуле:

S — тормозной путь,

Кт — тормозной коэффициент (для легкового автомобиля равен 1),

V — скорость автомобиля,

Kсц — коэффициент сцепления.

Понятия и пояснения

Тормозной путь — это путь, который проходит автомобиль с момента, когда сработал тормозной механизм до полной остановки автомобиля. На него влияют:

  • состояние и тип дорожного покрытия,
  • состояние шин автомобиля,
  • начальная скорость автомобиля,
  • масса автомобиля,
  • исправность тормозной системы.

Остановочный путь — путь с момента обнаружения опасности до полной остановки автомобиля. Понятно, что тормозной путь входит в остановочный. Кроме того в остановочный путь входят:

  1. путь, который проехал автомобиль с момента обнаружения опасности до нажатия на педаль тормоза;
  2. путь, пройденный автомобилем за время срабатывания тормозной системы.

Первый параметр зависит от множества факторов, определяющим из которых является времени реакции водителя. По результатам многочисленных экспериментов, оно может меняться от 0,3 до 1,5 секунды. В среднем можно считать время реакции водителя равное 1 секунде. Кроме этого существует понятие «нормативное время восприятия сложной ситуации» равное 0,8 секунды. Также установлено, что время реакции у женщин, при возникновении сложной дорожной ситуации может достигать 2,5-3 секунд, тогда как у мужчин 1,5-2 секунды. Кроме этого на время реакции влияет:

  • опыт водителя,
  • его эмоциональное состояние,
  • возраст,
  • время суток и погодные условия,
  • прием медикаментов,
  • состояние алкогольного или иного опьянения,
  • место возникновения опасной ситуации.

Время срабатывания тормозной системы зависит от ее типа и технического состояния. Тормозная система с гидравлическим приводом срабатывает за 0,2 – 0,3 секунды, с пневматическим за 0,5 –0,6 секунд.

Ваша оценка

[Оценок: 94 Средняя: 4.6]

Расчет тормозного пути Автор admin средний рейтинг 4.6/5 — 94 рейтинги пользователей

calculat.ru

что это и как рассчитать

Подробности
Категория: Блог о продаже автомобиля
Создано: 18 февраля 2020
Просмотров: 170

Всем водителям не следует забывать о том, что при нажатии на тормоз авто мгновенно не останавливается. Для этого нужно некоторое время, зависимое от многих факторов. Согласно ПДД, необходимо соблюдать безопасную дистанцию между своим и идущим впереди авто, чтобы при надобности успеть затормозить. Для расчета данной дистанции нужно знать, что такое тормозной путь. Кроме того, большинство людей путают остановочный и тормозной путь.

Тормозной путь: что это

Независимо от опытности и профессионализма водителя, за рулем случаются ситуации, когда нужно оперативно остановить авто:

  • На дороге неожиданно возникает животное или люди;
  • Возникает поломка авто;
  • Другие водители нарушают ПДД, создавая аварийную обстановку;
  • Происходят непредвиденные ситуации: препятствия, неровная дорога и т. д.

Чтобы остановить авто, водители пользуются тормозом. Тормозной путь транспорта – это отрезок, преодолеваемый машиной за время при срабатывании тормозной системы до достижения нулевой скорости.

Читайте также: Срочный выкуп аварийных авто

Факторы, влияющие на тормозной путь

Отрезок торможения разный, в зависимости от ситуации и условий. Факторы, воздействующие на данное расстояние, разделяются на 2 категории:

  1. Факторы, зависящие от автомобилиста;
  2. Факторы, не зависящие от водителя.

Состояние дорог и погода – это условия, на которые не влияет тот, кто сидит за рулем авто. Логично, что в снежную погоду необходимо больше времени, чтобы затормозить машину, чем в сухую. На отрезок торможения также влияет и покрытие трассы. На гладкой дороге, сделанной без включения камня, при торможении отрезок, который проедет транспорт, увеличится.

На заметку! На трассе, покрытой ямами, тормозной отрезок будет маленьким.

Это объясняется тем, что автомобилисты на плохих дорогах развивают небольшую скорость. Больше факторов, зависящих от водителя:

  • Устройство и работоспособность тормозной системы. Важно исправное состояние машины, включая и тормоза, не изношенность колодок, а также оптимальное давление в шинах;
  • Скорость. При малой скорости отрезок торможения уменьшается, и наоборот большая скорость приведет к увеличению расстояние, которое потребуется транспортному средству для остановки;
  • Используемые шины. Износ протектора должен быть оптимальным, а используемая резина должна быть по погоде. Довольно часто многие аварийные ситуации происходят из-за несвоевременной замены шин и из-за езды на некачественной, изношенной резине, которая не способна обеспечить необходимые характеристики и безопасность при езде;
  • Наличие ABS-системы. Такая система на сухом асфальте помогает быстро останавливать авто, в гололед она позволяет сохранять управление, но при этом увеличивается дистанция торможения;
  • Загрузка машины. Легче остановить легкий автомобиль, чем транспорт, который хорошо загружен. Нагруженная машина тормозит на более длинном отрезке;
  • Трезвость водителя. У трезвого человека более быстрая реакция на изменение ситуации на трассе, поэтому при необходимости он быстрее затормозит машину. Поэтому стоит отказываться от алкоголя, планируя сесть за руль. Это позволит избежать аварий и обезопасит людей;
  • Отсутствие отвлекающих моментов при езде. Часто медленная реакция водителя связывается с тем, что он не смотрит за дорогой и отвлекается. В настоящее время зачастую внимание отвлекает сотовый телефон и другие гаджеты, которые сводят людей с ума, не позволяя отказываться от них даже за рулем. Из-за замедленной водительской реакции увеличивается тормозной отрезок.
Формула для вычисления тормозного пути

Порой нужно рассчитывать тормозной отрезок, к примеру, в следующих условиях:

  • криминалистическая экспертиза;
  • испытания транспорта;
  • проверка работоспособности тормозной системы после ее доработки.

Чтобы произвести подобное вычисление, применяется формула:

Sторм = Кэ * V * V/(254 * Фс), где: S торм. – тормозной отрезок; V – скорость транспорта; Кэ – коэффициент торможения; Фс – коэффициент сцепления, который бывает различным. Так:

  • При мокрой дороге – 0,4;
  • При сухой – 0,7;
  • При гололеде – 0,1;
  • При снеге – 0,2.
Как тормозной путь зависит от дороги?

Коэффициент торможения – это постоянная величина, равняющаяся в основном единице. Рассмотрим пример. Автомобиль едет в летний сезон со скоростью 70 км/ч по сухому асфальту. Нужно вычислить тормозной путь.

S = 1*70*70/(254*0,7) = 28 метров – это и будет отрезок торможения.

Необходимо знать, что тормозной отрезок авто прямо пропорционален квадрату ее скорости. Так, при двойном увеличении скорости, к примеру, с 30 км/ч до 60 км/ч, отрезок торможения увеличился в 4 раза.

Отличие тормозного и остановочного пути

Остановочный и тормозной пути – это различные понятия, но их часто приравнивают и путают. Под остановочным путем понимается расстояние, прошедшее автомобилем после осознания водителем необходимости остановиться до достижения транспортом нулевой скорости. Тормозным путем называется отрезок, который проходит авто от срабатывания тормозов до остановки. Так, остановочный отрезок содержит не только отрезок торможения, но и отрезок, прошедший машиной, пока водитель реагировал на сложившуюся ситуацию.

Вычисление полного времени остановки и итогового тормозного пути

В итоге, величина данного пути содержит не просто дистанцию торможения, но и расстояние водительской реакции. Для вычисления расстояния, пройденного машиной за время водительской реакции, нужно использовать такую формулу:

S реакции = V/ 10*3, где V – скорость машины.

Так, общий тормозной путь станет равным сумме 2-х величин: тормозного пути и отрезок водительской реакции. S итог = Sреакции + Sторм.

Вспоминаем об авто, двигающемся в летнее время со скоростью 70 км/ч по сухой дороге, вычисляем расстояние реакции.

S реакции = 70/10 * 3 = 21 метр

Теперь, когда известно, что тормозной путь — 28 метров, а дистанция реакции – 21 метр, можно вычислить его общее значение: S итог = 28+ 21 = 49 метров

Под полным временем остановки понимается временный промежуток, за который автомобиль проходит общий путь торможения. Это время объединяет время, которое тратится на тормозной отрезок, и время водительской реакции.

Он вычисляется по формуле , где:

  • tp — время водительской реакции;
  • tc — время, когда срабатывает тормозной привод;
  • vo — начальная скорость торможения;
  • tH — время увеличения тормозных сил;
  • g — ускорение свободного падения;
  • Kэ — коэффициент эффективности торможения;
  • φ — коэффициент продольного сцепления колес авто с дорогой.

 

Важно! Общепринятой нормой времени водительской реакции считается 1 секунда. Общий остановочный отрезок включает расстояние водительской реакции и отрезок торможения. На данные величины воздействуют некоторые факторы. Для сокращения величины итогового значения нужно следовать скоростному режиму, проверять исправность авто, быть за рулем в трезвом состоянии и учитывать загрузку машины. Не стоит забывать, что многие ситуации и обстоятельства зависят от самого человека. Поэтому нужно самостоятельно проверять, не нарушать и быть внимательным, чтобы избегать неприятных случаев.

Понравилась статья?

Расскажи друзьям

Читайте также
Порядок и стоимость переоформления автомобиля

Транспортные средства юридических лиц и индивидуальных предпринимателей регистрируются по месту государственной регистрации этих юридических лиц и индивидуальных предпринимателей. Допускается регистрация транспортных средств юридических лиц по месту нахождения их филиалов, представительств и других обособленных подразделений.

Подробнее…
Особенности оформления купли-продажи автомобиля

Переход права собственности на транспортное средство предполагает выполнение некоторых бюрократических процедур и соблюдение ряда формальностей.

Подробнее…
Продал вторую машину-плати налоги

Многие автолюбители даже не подозревают, что, продав два или более авто в течение года, они обязаны подать декларацию в налоговую инспекцию. При этом, если Вы продали второй автомобиль дороже, чем купили, то обязаны заплатить налог с суммы продажи.

Подробнее…
Как продать машину без снятия с учета

Каким образом реализовать дорожное транспортное средство, не снимая с учета? Решение этой проблемы волнует многих автовладельцев.

Подробнее…
Договор купли-продажи автомобиля юридического лица физическому

На данный момент услугами рынка по продаже автомобилей пользуются не только частные лица, но и компании, так как они нуждаются в регулярном обновлении рабочих автомобилей.

Подробнее…
Как грамотно оформить договор купли-продажи автомобиля

При продаже автомобиля очень важно юридически грамотно оформить договор купли-продажи. Действующее законодательство регламентирует определённые правила проведения сделки, учитывая интересы и продавца и покупателя.

Подробнее…
Как избежать штрафа за тонировку авто

Если Вы любите затонировать свой автомобиль вкруговую, то данный текст именно для вас.

Как не получить штрафные санкции за подобное?

Подробнее…
Как продать автомобиль, полученный в наследство

После получения автомобиля по наследству, большинство людей задумываются о его продаже, причин этому достаточно много, возможно автомобиль старый, он вас не устраивает, вы хотите от него избавится или хотите вложить в какие-то внутренние инвестиции.

Подробнее…
Продажа авто при разводе, особенности и «подводные камни»

У нас часто спрашивают, как же продавать машину, если она была приобретена в браке, развод не за горами или уже состоялся.

Подробнее…
Договор купли – продажи и акт приема-передачи автотранспорта

Особо значимым документом при покупке автомобиля является как договор купли-продажи, так и акт приема-передачи автотранспорта. Имея под рукой пример его составления, можно справиться с оформлением акта купли-продажи, не обращаясь за помощью к юристу.

Подробнее…

avtovikyp.by

Как правильно вычислить дистанцию, тормозной и остановочный путь автомобиля: формулы расчета

Формулы расчета остановочного и тормозного пути, а также безопасной дистанции.

В теоретическом экзамене есть вопрос о среднем времени реакции водителя, правильным ответом на который является 1 секунда. Также в билетах ГИБДД имеется вопрос, связанный с безопасной дистанцией. Есть вопросы, касаемые торможения. Но, как говорится, теория – это теория, которая, увы, с практикой, как правило, не имеет ничего общего.

Во-первых, то, что вы учили в билетах, является теорией, основанной на усредненных значениях и различных исследованиях. Фактически же время реакции водителя, остановочный и тормозной путь зависят от многих факторов и не могут быть точно рассчитаны для всех случаев. Тем не менее каждый водитель должен уметь рассчитывать эти параметры хотя бы приблизительно. 

 

Тормозной путь автомобиля

Тормозной путь – это расстояние, которое будет пройдено автомобилем между контактом водителя с педалью тормоза и полной остановкой транспортного средства. Также стоит понимать различия между «нормальным торможением» и «экстренным торможением». В том числе не нужно забывать, что погодные условия влияют на тормозной путь. Если на дороге есть снег, тормозной путь, естественно, увеличивается. 

 

 

Вот формула расчета тормозного пути:

 

(Скорость в км / ч: 10) x (скорость в км / ч: 10) = тормозной путь в метрах

 

Пример расчета: представим, что вы едете со скоростью 50 км/ч по городу и подъезжаете к пешеходному переходу, по которому идут дети. Расчет: (50 км/ч : 10) х (50 км/ч : 10) = 25 (метров). Таким образом, тормозной путь вашей машины составляет 25 метров. Поэтому вы должны учитывать длину тормозного пути, чтобы спокойно своевременно начать тормозить и остановиться перед пешеходным переходом. 

 

Имейте в виду, что при экстренном торможении вы обычно нажимаете педаль тормоза полностью. В этом случае, как правило, тормозной путь сокращается вдвое. Вот формула тормозного пути при экстренном торможении:

 

 

(Скорость в км / ч: 10) x (скорость в км / ч: 10) / 2 = тормозной путь в метрах 

 

Пример расчета: вы едете по городу со скоростью 50 км/ч, и вдруг на дорогу выкатывается мяч, за ним бежит ребенок. Вам нужна экстренная остановка автомобиля. Расчет: (50 км/ч : 10) х (50 км/ч : 10)/2 = 12,5 (метров). Тормозной путь вашей машины при экстренном торможении составит 12,5 метра. 

 

 

Время и путь реакции водителя

Время реакции водителя – это время, которое пройдет с момента обнаружения водителем опасности на дороге до начала принятия мер по ее предотвращению.

 

Путь реакции водителя – это путь, который пройдет автомобиль с момента обнаружения водителем опасности на дороге до нажатия педали тормоза. 

Вот формула расчета пути, который пройдет автомобиль в момент реакции водителя на опасность: 

 

 

(Скорость в км / ч: 10) x 3 = путь реакции в метрах

 

Пример расчета: представим, что вы едете со скоростью 100 км/ч по проселочной дороге и внезапно на дорогу выбегает лось.  Расчет: (100 км/ч : 10) х 3 = 30 (метров). То есть, после того как вы среагируете на опасность на дороге, ваша машина проедет примерно 30 метров. Добавьте к этому тормозной путь автомобиля.

 

Внимание: эти правила не являются научно правильными формулами и дают только приблизительное значение!

 

Остановочный путь автомобиля

Остановочный путь – это расстояние, пройденное транспортным средством с момента обнаружения водителем опасности на дороге до полной остановки машины. 

 

 

Если вы хотите рассчитать остановочный путь автомобиля, вы должны добавить к тормозному пути автомобиля путь, пройденный за время реакции водителя. Вот как это можно сделать:

 

(Скорость в км / ч: 10) х 3 + (скорость в км / ч: 10) х (скорость в км / ч: 10)

Первое значение в выражении – это путь реакции водителя, пройденный автомобилем, пока водитель реагирует на опасность на дороге. Второе выражение – это формула расчета тормозного пути. Для того чтобы вычислить остановочный путь транспортного средства, необходимо оба результата сложить вместе. 

 

Пример расчета:  вы едете на своей машине со скоростью  50 км/ч. Расчет: (50 км/ч : 10) х 3 = 15 метров пути проедет машина при реагировании на опасность на дороге (50 км/ч : 10) x (50 км/ч : 10) = 25 метров составит тормозной путь автомобиля. В итоге, сложив оба значения, получаем, что остановочный путь транспортного средства составит 40 метров. 

 

Внимание: эти правила не являются научно правильными формулами и дают только приблизительное значение!

 

Дистанция

  • Три длины автомобиля. Любой, кто путешествует в городских условиях, должен соблюдать дистанцию ​​не менее 15 метров, или три длины автомобиля.
  • Половина спидометра: для безопасной дистанции за пределами населенных пунктов обращайте внимание на скорость автомобиля. Для того чтобы вычислить безопасную дистанцию, разделите на 2 текущую скорость, которую показывает спидометр. В итоге вы получите дистанцию до других автомобилей в метрах. Пример: на скорости 70 км/ч вы должны держаться до впереди идущего автомобиля на расстоянии не менее 35 метров. Причем это касается сухого асфальта в летнее время. 
  • Двойное расстояние: в случае плохой видимости или плохих дорожных условий вы должны удвоить безопасную дистанцию.

1gai.ru

Вот как можно рассчитать тормозной путь: Формула

Как рассчитать расстояние тормозного пути автомобиля.

 

Как быстро автомобиль ускоряется, наверное, знает большинство автовладельцев. Даже если вы не замеряли динамику разгона своей машины, вы наверняка смотрели заводские технические характеристики вашего авто, где обычно автопроизводитель указывает минимально возможное время разгона с 0-100 км/час. Но теперь вопрос: сколько времени нужно, чтобы остановить вашу машину? Вы знаете это? Уверены, что нет. Но, оказывается, рассчитать расстояние тормозного пути можно достаточно легко с помощью простой формулы. Мы расскажем вам, как это делается. 

 

Нет такой вещи во Вселенной или материи, которая может мгновенно остановиться. Также и любой автомобиль, когда вы нажимаете педаль тормоза, не сразу может остановиться. Дело в том, что для того чтобы автомобиль или любой объект в нашем мире остановился, необходимо, чтобы он потерял энергию, которая его движет. В результате у любого автомобиля есть тормозной путь, который он проезжает с момента нажатия педали тормоза до момента полной остановки. Это и есть тормозное расстояние машины.

 

Но на самом деле тормозной путь любого авто зависит не только от его характеристик и тормозной системы, но и от реакции водителя при нажатии педали тормоза. Ведь для того чтобы принять решение о необходимости торможения и нажать педаль тормоза, требуется время, которое хоть и минимально, но достаточно, чтобы машина успела проехать немаленький путь. Особенно это важно при большой скорости движения, где за какие-то доли секунды автомобиль проезжает приличное расстояние. Итак, в итоге, чтобы рассчитать реальную длину тормозного пути, нужно учитывать не только время и расстояние, пройденное автомобилем с момента нажатия водителем педали тормоза до момента остановки машины, но и время, необходимое для принятия решения о торможении. Дело в том, что при принятии решения о торможении мы тратим драгоценные секунды. Вот пример:

 

  • Время отклика: Прежде чем водитель нажмет педаль тормоза, он должен оценить дорожную ситуацию и определить, необходимо ли торможение. Также нужно понять, какое необходимо торможение – полная остановка автомобиля или простое снижение скорости. Обычно, согласно многочисленным исследованиям, большинству водителей для этого требуется около 0,1 секунды. 
  • Время, необходимое для нажатия педали тормоза: После того, как водитель понял, что должен тормозить, необходимо еще примерно 0,8 секунды, для того чтобы переместить ногу с педали газа на педаль тормоза и нажать ее.  

 

Кроме того, даже при нажатии педали тормоза есть еще небольшая потеря времени, связанная с тем, что при нажатии педали тормоза автомобиль, как правило, не начинает резко тормозить. А для того чтобы машина реально начала резко снижать скорость, надо усилить давление на педаль тормоза (пороговое время, необходимое для требуемого тормозного давления в тормозной системе). Также у всех автомобилей разное время отклика на нажатую педаль тормоза. Здесь все, конечно, зависит от конструкции тормозной системы и наличия различной электроники, контролирующей тормоза автомобиля.

 

Смотрите также: Полный привод оказался лучше при торможении, чем привод на два колеса: Видео

 

Вы не поверите, но для того чтобы машина реально начала тормозить после нажатия педали тормоза, необходима еще почти 1 секунда времени. Вы представляете, как это много при движении на большой скорости? За эту лишнюю секунду вы можете проехать очень большой путь. 

 

Что такое формула тормозного пути?

В общем, торможение автомобиля делится на два вида. Например, есть нормальное торможение, а есть экстренное, когда вам нужно резко остановить машину, чтобы избежать аварии.

 

При торможении в повседневной жизни, допустим, если вы хотите остановить автомобиль на светофоре, вы обычно нажимаете педаль тормоза намного плавнее и мягче, чем при необходимости полностью остановить автомобиль на парковке во дворе. В этом случае вы не применяете в машине максимальное тормозное усилие. При таком плавном и мягком торможении, как правило, тормозной путь (тормозное расстояние) увеличивается. Примерное расстояние тормозного пути при нормальном торможении можно рассчитать по следующей простой формуле:

 

(Скорость в км/ч : 10) x (скорость в км/ч : 10) = тормозной путь в метрах

 

При экстренном торможении педаль тормоза, как правило, нажата целиком и с полной силой. Из-за более высокой силы торможения обычно тормозной путь машины сокращается примерно в 2 раза. Поэтому длину тормозного пути можно также вычислить по следующей формуле:

 

(Скорость в км/ч : 10) x (скорость в км/ч : 10) / 2 = тормозной путь в метрах

 

Внимание: Вычисляемый по этим формулам тормозной путь является лишь приблизительным значением и подсказкой для водителей. На самом деле в реальности тормозной путь может быть как меньше, так и больше. Ведь расстояние тормозного пути зависит от навыков и опыта вождения водителя, от технической исправности автомобиля, его конструкции, марки, модели, состояния дорог, состояния протектора резины и многих других факторов, которые напрямую влияют на длину тормозного пути. Но благодаря этим формулам вы примерно сможете высчитать среднюю длину тормозного пути машины при определенной скорости движения. Это позволит вам скорректировать ваш стиль управления автомобилем, а также станет хорошим пособием для водителей-новичков. 

 

Как рассчитать полное время остановки и итоговый тормозной путь?

 

Как мы уже сказали, чтобы рассчитать весь тормозной путь, нужно учитывать потерю времени при принятии водителем решения о торможении (то есть время реакции водителя). Для этого нужно использовать другую формулу, которая обеспечивает более точный приблизительный расчет тормозного расстояния, которое проедет автомобиль в момент принятия решения о необходимости остановки. Вот эта формула:

 

(Скорость в км/ч : 10) x 3 = путь реакции в метрах

 

В итоге, сделав вычисление по вышеуказанным формулам, вы можете вычислить приблизительный итоговый тормозной путь вашего автомобиля при любой скорости движения. Вот пример. Если вы управляете своим автомобилем со скоростью 50 км/ч, то с помощью приведенных формул вычислите следующие значения:

 

  • Тормозной путь при принятии решения о торможении на этой скорости (реакция на дорожную ситуацию + принятие решения о торможении + время, необходимое для перемещения ноги с педали газа на педаль тормоза, а также время отклика тормозной системы на нажатую педаль тормоза) составит где-то (50/10) х 3 = 15 метров. То есть пока вы будете принимать решение о торможении при скорости в 50 км/ч, ваша машина проедет 15 метров. 
  • Тормозной путь при нормальном торможении (с момента нажатия педали тормоза до момента остановки машины) составит около (50/10) х (50/10) = 25 метров.  
  • При экстренном торможении тормозной путь, как мы уже отметили, сокращается примерно в два раза. Соответственно, расчет тормозного расстояния автомобиля, который движется со скоростью 50 км/ч, будет выглядеть следующим образом: (50/10) x (50/10) / 2 = 12,5 метров.
  • В результате теперь мы можем вычислить реальный итоговый тормозной путь автомобиля. Так, при нормальном (не резком, а обычном) торможении итоговый тормозной путь составит около 40 метров. При экстренном торможении – не менее 28 метров. 

 

Примечание: Обратите внимание, что если скорость автомобиля будет выше всего в два раза, его итоговый тормозной путь увеличится в четыре раза!!!

 

Смотрите также: Основные принципы работы тормозного механизма автомобиля [Принцип работы и элементы тормозной системы]

 

То есть мнение о том, что при увеличении скорости автомобиля в два раза тормозной путь увеличивается только в два раза, – это чистый воды миф среди многих автолюбителей. Так что имейте это в виду, когда садитесь за руль. Самое удивительное, что об этом не знают даже многие опытные водители. 

 

Пример расчета тормозных и остановочных расстояний

Скорость, в км / ч

Путь, пройденный автомобилем

во время реакции водителя, в метрах

Тормозное расстояние, в метрах

(с момента нажатия педали тормоза

до полной остановки машины)

Итоговый тормозной путь, в метрах

25

7,5

6,25

13,75

50

15

25

40

100

30

100

130

150

45

225

265

200

60

400

460

 

Какие факторы влияют на торможение и тормозной путь?

 

Решающим значением для длины тормозного пути, конечно же, является скорость автомобиля, с которой он движется по дороге. Также на тормозной путь влияет качество установленной на машину тормозной системы. В том числе важную роль, несомненно, играет и состояние дороги (снег, лед, качество асфальта/бетона, трещины в дорожном покрытии, листья, лужи и т. п.). И само собой, не стоит забывать о состоянии шин автомобиля. Ведь в определенных случаях изношенная резина сильно увеличит тормозной путь автомобиля, так как не сможет передавать нормальную тормозную способность дорожному покрытию в отличие от новых шин, имеющих нормальное сцепление с дорогой. 

Также ясно, что на мокрой поверхности тормозное расстояние машины больше, чем на сухом асфальте. 

 

Не стоит забывать и об уровне подготовки водителя. Особенно важна, как мы узнали, для итогового тормозного пути скорость реакции водителя на дорожную ситуацию, требующую остановки автомобиля. Но скорость реакции за рулем зависит не только от опыта вождения. Например, знаете ли вы, что когда вы садитесь за руль в сонном состоянии (не выспались, устали или долго находились за рулем), то скорость реакции может замедлиться почти в два раза по сравнению со скоростью реакции хорошо отдохнувшего водителя.  

В целом же на скорость принятия решения за рулем (скорость реакции) влияет много факторов: возраст водителя, алкогольное или похмельное состояние, употребление определенных медикаментов и в целом состояние здоровья. Так, при многих хронических заболеваниях скорость реакции многих водителей существенно снижается. Следовательно, все эти факторы серьезно влияют на тормозной путь автомобиля. 

 

Смотрите также: Тормозной путь автомобиля: Все что нужно знать

 

То же самое касается и отвлечения внимания из-за смартфонов, которыми так любят пользоваться за рулем многие водители, несмотря на строгий запрет согласно нашему действующему законодательству.

 

Как мы уже сказали, на тормозной путь также влияет время отклика тормозной системы автомобиля на нажатую педаль тормоза. Особенно это касается старых автомобилей. Современные же, как правило, оснащены уже новым поколением тормозов, которые мгновенно активируются за счет максимального тормозного давления, как только вы резко ударите ногой по педали тормоза (например, при экстренном торможении). Эта технология позволила существенно сократить итоговый тормозной путь современных машин. 

 

Как повысить безопасность при управлении автомобилем?

 

Не зря основное правило вождения гласит о том, что водитель должен держать на дороге достаточную дистанцию до других автомобилей, чтобы оставалось пространство для экстренного торможения и для того, чтобы не спровоцировать ДТП. Но, с другой стороны, вы не должны держать дистанцию между автомобилями слишком большой. Помните, что все должно быть в меру. Вот некоторые правила вождения от экспертов:

 

  • В городском движении: Держите расстояние до других автомобилей около 15 метров. 
  • На автомагистралях, шоссе и проселочных дорогах: При скорости движения около 100 км/ч держите дистанцию примерно 50 метров. При плохой видимости или на скользкой дороге дистанция до других машин должна быть увеличена в два раза. Например, при скорости в 100 км/ч на скользкой дороге держите расстояние до впереди идущей машины минимум в 100 метров.  

1gai.ru

Как найти скорость торможения формула

Известно, что грузовой автомобиль массой пять тысяч килограмм движется по горизонтальному пути со скоростью семьдесят два километра в час (20 метров в секунду).
Необходимо: определить силу и время торможения автомобиля, если тормозной путь составил пять метров.

Дано: m=5000 кг; v=20 м/сек; s=5 м
Найти: F-?; t-?

Исходя из того, что работа силы торможения численно равна изменению кинетической энергии движущегося автомобиля , получаем формулу для определения силы торможения

Подставив в формулу численные значения, рассчитаем силу торможения грузового автомобиля

н

Из формулы , при условии, что vt=0: , где , получаем формулу времени торможения

Время торможения автомобиля

сек

Ответ: сила торможения автомобиля составила двести тысяч ньютон, время торможения равно половине секунды.

Тормозной путь – расстояние, которое потребуется автомобилю, чтобы полностью остановиться с момента начала работы системы торможения.

В обиходе этот термин часто путают с остановочным, однако тормозной и остановочный путь – разные понятия. В последнем случае учитывается расстояние, прошедшее с момента осознания водителем необходимости торможения до скорости 0 км/ч. Тормозной путь – часть остановочного.

От чего зависит тормозной путь

Рассматриваемый показатель не является постоянной величиной и может варьировать по ряду причин. Все факторы, влияющие на путь торможения, можно разделить на две большие группы: зависящие от водителя и независящие от водителя. К числу причин, не зависящих от человека за рулем, относят:

Несложно догадаться, что в дождь, снег или гололед расстояние, которое потребуется для остановки автомобиля, будет большим, чем на сухом асфальте. Торможение окажется длительным и при движении по гладкому асфальту, в который не была добавлена каменная крошка. Здесь колесам не за что зацепиться, в отличие от шершавых покрытий.

На заметку: стоит заметить, что плохое качество дороги (ямы, выбоины) не приводит к удлинению расстояния, необходимого для остановки. Здесь играет роль человеческий фактор. Пытаясь сберечь подвеску, водители редко развивают высокую скорость на подобных дорогах. Соответственно, путь торможения здесь минимален.

Факторы, зависящие от водителя или владельца авто:

  • состояние тормозов;
  • устройство системы;
  • наличие ABS;
  • вид покрышек;
  • загруженность ТС;
  • скорость движения.

Тот факт, что длина тормозного пути автомобиля напрямую зависит от исправности системы торможения, не требует доказательств. Машина с неработающим тормозным контуром или изношенными колодками никогда не сможет остановиться также быстро, как исправное ТС.

От устройства тормозных агрегатов зависит многое. Современные машины, оснащенные задними дисковыми тормозами и системами помощи при торможении, имеют гораздо лучшее сцепление с дорогой и короткий отрезок торможения.

В свою очередь, наличие EBD с ABS не всегда способствует сокращению расстояния, необходимого для остановки. На сухом твердом покрытии, где блокировка колес наступает только при очень интенсивном торможении, система действительно сокращает тормозной путь. Однако на голом льду «умный» электронный помощник начинает сбрасывать тормозное усилие даже при легком нажатии на педаль тормоза. При этом авто сохраняет управляемость, однако путь его торможения значительно увеличивается.

От чего зависит скорость замедления? Разумеется, от вида покрышек. Так, на голом, пусть и промороженном асфальте, а также в снежной каше, лучше всего тормозят т. н. «липучки» — зимние покрышки, не оснащенные шипами. В свою очередь, в гололед и на заснеженных дорогах наиболее эффективной является ошипованная «резина».
» alt=»»>
Немаловажным фактором, влияющим на величину остановочного отрезка, является скорость и загруженность машины.

Понятно, что легковесный автомобиль при скорости 60 км/ч остановится быстрее, чем грузовик, загруженный под завязку и движущийся со скоростью 80-100 км/ч. Последнему не позволит быстро остановиться слишком высокая для него скорость и инерция.

Когда и как производится замер

Расчет тормозного пути может потребоваться в следующих случаях:

  • технические испытания транспортного средства;
  • проверка возможностей машины после доработки тормозов;
  • криминалистическая экспертиза.

Как правило, при расчете используют формулу S=Кэ*V*V/(254*Фс). Здесь S – тормозной путь; Кэ – тормозной коэффициент; V₀ — скорость на момент начала торможения; Фс – коэффициент сцепления с покрытием.

Коэффициент сцепления с дорогой изменяется в зависимости от состояния покрытия и определяется по следующей таблице:

Состояние дорогиФс
Сухая0.7
Мокрая0.4
Снег0.2
Лед0.1

Коэффициент Кэ является статической величиной и составляет единицу для всех наиболее распространенных легковых транспортных средств.

Пример: как рассчитать тормозной путь автомобиля при цифре 60 км/ч на спидометре в дождь? Дано: скорость 60 км/ч, тормозной коэффициент – 1, коэффициент сцепления – 0.4. Считаем: 1*60*60/(254*0.4). В итоге получаем цифру 35.4, что и является длиной тормозного пути в метрах.

В таблице указано сколько метров машина будет продолжать движение до полной остановки. Следует учитывать, что в расчет не берутся никакие иные показатели (повороты, выбоины на дороге, встречный поток и т.д.). Сомнительно, что в реальных условиях на обледенелой дороге, автомобиль сможет проскользить километр и не встретить столб или отбойник.

СкоростьСухоДождьСнегЛед
км/чметры
6020,235,470,8141,7
7027,548,296,4192,9
8035,962,9125,9251,9
9045,579,7159,4318,8
10056,298,4196,8393,7
11068119238,1476,3
12080,9141,7283,4566,9
13095166,3332,6665,3
140110,2192,9385,8771,6
150126,5221,4442,9885,8
160143,9251,9503,91007,8
170162,5284,4568,81137,7
180182,2318,8637,71275,5
190203355,3710,61421,2
200224,9393,7787,41574,8

Мы нашли интересный калькулятор, который не только рассчитывает показатель в зависимости от скорости и состояния дороги, но и наглядно показывает весь процесс. Находится здесь.

Как увеличить интенсивность замедления

Из вышесказанного стало понятно, что называется тормозным путем и от чего зависит этот показатель. Однако возможно ли сократить расстояние, которое необходимо для остановки автомобиля? Возможно! Для этого существует два пути – поведенческий и технический. Идеально, если водитель сочетает оба способа.

  1. Поведенческий метод – сократить тормозной путь можно, если выбирать небольшую скорость движения на скользких и мокрых дорогах, учитывать степень загруженности машины, грамотно рассчитать тормозные возможности авто в зависимости от его состояния и модельного года. Так, «москвич» 1985 года разработки не сможет тормозить столь же эффективно, как современный «Hyundai Solaris», не говоря уж о более респектабельных и технологичных моделях.
  2. Технический метод – метод усиления тормозных возможностей, основанный на повышении мощности тормозной системы и использовании вспомогательных механизмов. Производители современных ТС активно применяют такие способы улучшения тормозов, оснащая свою продукцию антиблокировочными системами, системами помощи при торможении, используя более эффективные тормозные диски, колодки.

Следует помнить, что сокращение времени, необходимого для остановки – один из способов обеспечения безопасности поездки. Поэтому каждый водитель должен постоянно следить за техническим состоянием своего «железного коня», своевременно обслуживать и ремонтировать систему торможения. Помимо этого, важно выбирать скорость движения с учетом окружающей обстановки: времени суток, состояния дороги, модели автомобиля и прочее.

При торможении на автомобиль действует сила трения скольжения, поэтому по 2 закону Ньютона: µ mg = ma , отсюда а = µg. Путь до полной остановки рассчитывается по формуле: S = v 2 / 2 a . Тогда тормозной путь равен : S = v 2 / 2µ g . Остановочный путь равен сумме пути автомобиля во время реакции водителя (равномерное движение) и тормозного пути: S = vt + ( v 2 / 2µ g ) .

При повороте на тело также действует сила трения, но тело движется по окружности, поэтому 2 закон Ньютона будет иметь вид: µ mg = mv 2 / R . Тогда радиус поворота равен: R = v 2 / µ g .

Не вписался в поворот

1. Вычислить остановочный путь автомобиля для начальной его скорости 72 км/ч , если он замедляется с постоянным ускорением 6 м/с 2 , а время реакции водителя составляет 1с.

2. Шофер автомобиля, едущего со скоростью 60 км/ч, внезапно видит перед собой группой школьников, собирающейся перейти дорогу. Как поступить целесообразнее: затормозить или повернуть?

Для домашнего задания:

1. На трассе за чертой города скорость автомобиля 110 км/ч. Включен дальний свет. Дорогу перебегает заяц. Какова его судьба?

2. На горизонтальной дороге автомобиль делает поворот радиусом 16 м. Какова наибольшая скорость, которую может развить автомобиль, чтобы его не занесло, если коэффициент трения скольжения колес о дорогу равен 0,4?

3. Шофер автомобиля, едущего со скоростью 60 км/ч, внезапно видит перед собой на расстоянии 40 м широкую стену. Что ему выгоднее: затормозить или повернуть?

4. Выясните технические характеристики вашего семейного автомобиля (если в семье нет автомобиля – выберите в интернете понравившуюся модель и изучите ее). Какие технические средства безопасности пассажиров есть в автомобиле, выясните наличие антиблокировочная тормозная система и узнайте мнение водителя о ее роли.

5. При возможности практически выясните глубину зоны видимости при ближнем свете, входя в эту зону в темной одежде и со светоотражателем – например со знаком аварийной остановки в руках. Автомобиль стоит.

l2rv.ru

Тормозной путь, формула — Лада мастер

Беспечность выглядит эффектно только в хорошо продуманных сценах из боевиков и детективов. На самом же деле, большинство водителей даже не представляют, о какой опасности идёт речь, когда говорят о соблюдении дистанции и о превышении скорости. Многие ли падали с трехметровой высоты плашмя на бетонный пол? Едва ли. А на самом деле, точно такую же нагрузку будет испытывать человек в автомобиле при наезде на неподвижное препятствие на скорости… всего 28 км/ч.

Содержание:

Зачем знать длину тормозного пути

Раз уж мы начали с расчётов, говоря о длине тормозного пути движущегося автомобиля, используем простую физическую формулу, известную каждому школьнику. Её используют для вычисления перехода энергии падения в кинетическую энергию конце пути (mgh=mVx2/2). Отсюда получаем, что при скорости около 30 км/ч тело получает удар, равный падению с высоты три метра. Соответственно, при движении на скорости 60 км/ч сила удара будет равна падению с высоты 15м, а уже на скорости 90 км/ч — с высоты около 32 м, 120 км/ч — это уже высота 55 метров.

Даже учитывая, что в автомобиле срабатывает подушка безопасности, выжить при лобовом ударе на скорости 60 км/ч шансов очень мало. Это примерная высота хрущевки. Отважится ли кто-то прыгнуть с крыши пятиэтажки, обвязавшись надувными подушками? Едва ли. А что говорить о скорости в 90 км/ч, удар при которой равносилен падению с высоты десятиэтажного дома? А с высоты 55 метров? Шансов выжить никаких, и это даже при условии, что подушка безопасности сработает безукоризненно.

Эмпирическая формула расчёта тормозного пути

Имея отличный водительский глазомер и достаточный опыт, каждый сможет определить расстояние до объекта на глаз, хотя бы примерно. Водительский опыт показывает, что для мгновенного вычисления длины тормозного пути по скорости, необходимо просто бросить взгляд на спидометр, оценить расстояние до препятствия, тогда тормозной путь будет равен половине числа, которое показывает спидометр. То есть, исходя из эмпирической формулы расчёта длины тормозного пути, безопасная дистанция до любого объекта будет равна мгновенной скорости, разделённой пополам. Практически так же производят расчёт скорости автомобиля по тормозному пути.

При этом нужно учитывать такое понятие, как остановочный путь, это термин экспертов дорожной полиции и он учитывает не только сам по себе тормозной путь, но и скорость реакции, а также время реагирования системы тормозов. В принципе — это расстояние до абсолютной остановки машины от того момента, когда водитель зафиксировал препятствие. Естественно, остановочный путь всегда больше тормозного, поскольку средняя скорость реакции здорового и трезвого водителя около 0,8 с, а тормозная система срабатывает ещё за 0,2-0,3 с. Следовательно, до полной остановки машины пройдёт ещё 1,1 с, а на скорости 60 км/ч автомобиль проходит 16,6 метров за одну секунду. Почти семнадцать метров, которые неминуемо будут добавляться к длине тормозного пути и которые редко учитываются большинством водителей. Вот именно поэтому необходимо серьёзно отнестись хотя бы к теоретическому вычислению длины тормозного пути.

Что нужно для расчёта тормозного пути

Чтобы вычислить тормозной путь формула которого указана на рисунке с пояснениями, мало знать моментальные сухие данные.

Теоретически, для оценки тормозных характеристик машины необходимо использовать массу данных:

  • длину тормозного пути;
  • минимальное время, за которое тормозная система сработает;
  • диапазон изменения тoрмозных усилий;
  • алгоритм изменения тoрмозных усилий;
  • производительность тормозов в зависимoсти от нагрева;
  • качество дорожного покрытия;
  • эффективность подвески автомобиля;
  • степень износа и тип покрышек.

Здесь нужно учитывать целый ряд моментов. К примеру, эффективность работы тормозной системы в каждом автомобиле может быть разной и это само собой разумеется. Гидравлическая система тормозов даёт задержку минимум 0,2-03 с, а пневматика, установленная на большинстве грузовиков и автобусов и того больше, до 0,6 с. Кроме этого, есть такое понятие, как нарастание тормозного усилия с нуля до максимального значения и это также отбирает от 0,4 до 0,6 с, при этом влияние скорости движения на длину тормозного пути в этом случае увеличивается в квадрате, то есть при увеличении скорости в два раза, тормозной путь будет вчетверо длиннее.

Дополнительные составляющие тормозного пути

При вычислении эффективности тормозов очень большое значение имеет характеристика подвески и состояние шин. При чем тут подвеска? Очень просто. У нас под колёсами довольно редко встречается идеально ровный асфальт, а именно подвеска, точнее, амортизаторы, рессоры, торсионы и пружины как раз и прижимают колеса к поверхности, делая торможение и управление максимально эффективным. Если амортизатор неисправен, колеса подпрыгивают на ухабах и о полном контакте с покрытием не может быть и речи.

Давайте к этому прибавим кoэффициент сцепления резины с дорoгой — здесь огромное значение имеет состояние дороги, тип покрышки (зима  или лето), рисунок протектора, геометрия, износ прoтектора и качество резиноматериала. Тесты показали, что на одном и том же автомобиле, но с разными покрышками, длина тормозного пути может изменяться до трёх-пяти метров, а о качестве пoкрытия и говорить нечего. Попробуйте сравнить тoрможение на сухом асфальте и на льду.

Как видим, факторов, влияющих на тормозной путь, а тем более на остановочный, достаточно много, поэтому предельная концентрация внимания за рулём — это гарантия безопасной езды. Проверяйте тормоза вовремя, не говорите по телефону за рулём и пусть все ваши дороги будут добрыми!

ladamaster.com

Как найти путь торможения физика

Тормозной путь — расстояние, которое проходит транспортное средство с момента срабатывания тормозной системы до полной остановки.

Протяжённость тормозного пути зависит от скорости, состояния проезжей части, шин, погодных условий. Особое влияние на протяжённость тормозного пути оказывает эффективность тормозной системы (ТС). Она складывается из технологических особенностей узлов ТС — «Электронных помощников», логики их работы, диаметра тормозных дисков, материала тормозных колодок, принудительной вентиляции и других параметров.

А) модуль силы трения, действующей

Б) тормозной путь грузовика

А) Сила трения при торможении грузовика равна . Это соответствует варианту ответа 1.

Б) Грузовик тормозит за счет силы трения, то есть с ускорением

Время торможения до полной остановки можно найти как

.

Тогда тормозной путь будет равен

и подставляя вместо t и a найденные выражения, получаем:

.

Калькулятор тормозного пути позволит оценить тормозной путь автомобиля, движущегося с заданной скоростью. Для использования укажите тип дорожного покрытия, на котором тормозит автомобиль и скорость, при которой начинается торможение. Калькулятор рассчитает сколько метров пройдет автомобиль при торможении.

Калькулятор тормозного пути

Формула тормозного пути

Формула для нахождения тормозного пути

Формула для нахождения тормозного пути применяется в подразделениях ГИБДД. Именно она используется в нашем калькуляторе. В этой формуле:

S — тормозной путь,

Кт — тормозной коэффициент (для легкового автомобиля равен 1),

V — скорость автомобиля,

Kсц — коэффициент сцепления.

Понятия и пояснения

Тормозной путь — это путь, который проходит автомобиль с момента, когда сработал тормозной механизм до полной остановки автомобиля. На него влияют:

  • состояние и тип дорожного покрытия,
  • состояние шин автомобиля,
  • начальная скорость автомобиля,
  • масса автомобиля,
  • исправность тормозной системы.

Остановочный путь — путь с момента обнаружения опасности до полной остановки автомобиля. Понятно, что тормозной путь входит в остановочный. Кроме того в остановочный путь входят:

  1. путь, который проехал автомобиль с момента обнаружения опасности до нажатия на педаль тормоза;
  2. путь, пройденный автомобилем за время срабатывания тормозной системы.

Первый параметр зависит от множества факторов, определяющим из которых является времени реакции водителя. По результатам многочисленных экспериментов, оно может меняться от 0,3 до 1,5 секунды. В среднем можно считать время реакции водителя равное 1 секунде. Кроме этого существует понятие «нормативное время восприятия сложной ситуации» равное 0,8 секунды. Также установлено, что время реакции у женщин, при возникновении сложной дорожной ситуации может достигать 2,5-3 секунд, тогда как у мужчин 1,5-2 секунды. Кроме этого на время реакции влияет:

  • опыт водителя,
  • его эмоциональное состояние,
  • возраст,
  • время суток и погодные условия,
  • прием медикаментов,
  • состояние алкогольного или иного опьянения,
  • место возникновения опасной ситуации.

Время срабатывания тормозной системы зависит от ее типа и технического состояния. Тормозная система с гидравлическим приводом срабатывает за 0,2 – 0,3 секунды, с пневматическим за 0,5 –0,6 секунд.

l2rv.ru

Как определить тормозной путь формула физика

Сухой асфальт

На сухом асфальте коэффициент сцепления шины составляет 0,7–0,8. Это отличный показатель.

Мокрый асфальт, лёд, снег

На мокром асфальте коэффициент сцепления составляет 0,4–0,5.

Виды торможения

Сначала рассмотрим способы:

  • газ-тормоз;
  • ступенчатый, с понижением усилия;
  • ступенчатый, с повышением усилия;
  • прерывистый;
  • силовое торможение мотором;
  • торможение силовым агрегатом.

А теперь рассмотрим виды:

  1. Аварийное. Аварийное торможение используется тогда, когда обычные способы не приносят необходимых результатов.
  2. Стояночное. Для торможения применяется ручной тормоз. Стояночное торможение применяется для фиксации транспортного средства в состоянии покоя.
  3. Экстренное. Такой способ используется при возникновении экстренной ситуации. Такой способ позволяет максимально быстро замедлить машину.
  4. Служебное. Это стандартный способ. Существует два варианта:
  • до полной остановки машины — применяется для полной остановки машины;
  • частичное — этот способ используется для снижения скорости.

Экстренное торможение

Как правило, экстренное торможение используется в старых машинах, которые не оборудованы современными системами безопасности (ESP, ABS и т. д.).

Определение скорости авто с помощью тормозного пути

Проводить расчёт по формуле достаточно сложно. Для определения скорости машины можете воспользоваться специальными онлайн-калькуляторами. Найти такой онлайн-калькулятор можно в поисковой системе.

Онлайн-калькуляторы разработаны с учётом всех требований. В них учтены все данные и формулы.

Вам нужно только вести такие данные:

  • длина следа торможения;
  • вид дорожного покрытия;
  • степень загрузки транспортного средства;
  • тип автомобиля;
  • скорость движения.

Далее, всю работу за вас сделает онлайн-калькулятор.

А теперь рассмотрим формулу для определения скорости движения. Формула: 0.5 х t3 х j + √2Sю х j.

Описание:

  • Sю — это длина следа;
  • j — этот символ обозначает замедление транспортного средства при торможении;
  • t3 — это нарастание замедления машины;
  • Va — начальная скорость машины.

Отличие тормозного пути от остановочного

Рассмотрим подробнее эти два понятия.

  1. Тормозной. Это расстояние, проходимое машиной с момента нажатия на педаль. Он является частью остановочного пути.
  2. Остановочный. Это расстояние, проходимое машиной с момента обнаружения опасности.

Определить тормозной путь автомобиля можно двумя способами:

  • провести расчёты по специальной формуле;
  • воспользоваться онлайн-калькуляторами, которые можно найти на профильных сайтах.

На тормозной путь влияет большое количество факторов (дорожное покрытие, состояние транспортного средства, погодные условия, навыки водителя, способ торможения, протектор шин).

Не нашли ответа на свой вопрос? Узнайте, как решить именно Вашу проблему — позвоните прямо сейчас:

+7 (499) 455-03-75 (Москва)
+7 (812) 407-26-30 (Санкт-Петербург)

Может случиться так, что от длины тормозного пути будет зависеть целостность кузова автомобиля и сохранность его пассажиров. Автомобиль на скорости просто не может резко замереть после нажатия на тормоз, даже если на нем стоят качественные покрышки и эффективная система торможения. После того, как нажата педаль тормоза, машина в любом случае преодолевает некоторое расстояние, и называется это расстояние — тормозной путь.

Водителю необходимо постоянно рассчитывать длину тормозного пути в соответствии с одним из правил по безопасности движения, которое говорит о том, что путь торможения должен быть меньше, чем расстояние до помехи.

В данной ситуации, все зависит от реакции и умения водителя, чем раньше он нажмет на тормоз и правильнее рассчитает длину тормозного пути, тем раньше, и успешнее авто затормозит.

Тормозной путь автомобиля при скорости 60 км/ч

Длина остановочного пути также зависит не только от водителя, но и от других сопутствующих факторов: от качества дороги, скорости движения, погодных условий, состояния тормозной системы, устройства тормозной системы, шин автомобиля и многих других.

Обратите внимание, что вес легкового автомобиля не влияет на длину тормозного пути. Это связано с тем, что вес автомобиля увеличивает инертность автомобиля при выполнении торможения, препятствуя при этом торможению, но увеличивает сцепление шин с дорогой благодаря увеличенной массе авто.

Эти физические свойства компенсируют друг друга, при этом практически не оказывая влияние на длину тормозного пути.

Скорость торможения напрямую зависит от способа торможения. Резкий тормоз до упора, приведет к заносу или движению машины юзом (если машина не оборудована системой ABS).

Постепенное нажатие на педаль применяется когда на дороге хорошая видимость и спокойная обстановка, оно не подходит для экстренных ситуаций. При прерывистом нажатии можно потерять управляемость, но зато быстро остановиться. Также возможно ступенчатое нажатие (схоже по эффекту с системой АБС).

Существуют специальные формулы, которые позволяют определить длину тормозного пути. Мы попробуем просчитать формулу по разным условиям, в зависимости от типа дорожного покрытия.

Формула для определения тормозного пути
Тормозной путь на сухом асфальте

Вспоминаем уроки физики, где ? – это коэффициент трения, g – ускорение свободного падения, а v – скорость движения машины в метрах в секунду.

Ситуация следующая: едет водитель на автомобиле Lada скорость которого 60 км/час. Буквально в 70 метрах идет женщина преклонного возраста, которая забыв о правилах безопасности спешно догоняет маршрутное такси (стандартная ситуация для России).

Воспользуемся этой самой формулой: 60 км/ч = 16,7 м/сек. У сухого асфальта коэффициент трения равняется 0,7 , g – 9,8 м/с. На самом деле, в зависимости от состава асфальта, он равен от 0.5 до 0.8, но всё же возьмем усредненное значение.

Полученный по формуле результат 20,25 метров. Естественно, что данное значение уместно лишь для идеальных условий, когда на машину установлена качественная резина и тормозные колодки, тормозная система исправна, при торможении вы не уходите в юз и не теряете управление, от множества других идеализированных факторов, которые не встречаются в природе.

Также для перепроверки результата, существует еще одна формула определения тормозного пути:

S = Кэ * V * V / (254 * Фс) , где Кэ – тормозной коэффициент, для легковых авто он равняется единице; Фс – коэффициент сцепления с покрытием 0,7 (для асфальта).

Подставляем скорость движения транспортного средства в км/ч.

Получается, что тормозной путь 20 метров для скорости 60 км/ч, (для идеальных условий), в том случае если торможение будет резким и без юза.

Тормозной путь на покрытии: снег, лед, мокрый асфальт

Коэффициент сцепления помогает обозначить длину остановочного пути при разных дорожных условиях. Коэффициенты для разных дорожных покрытий:

  • Сухой асфальт – 0,7
  • Мокрый асфальт – 0,4
  • Укатанный снег – 0,2

Попробуем подставить эти значения в формулы, и найдем значения длины тормозного пути для дорожного покрытия в разное время года и при разных погодных условиях:

  • Мокрый асфальт – 35,4 метра
  • Укатанный снег – 70,8 метра
  • Лед – 141,6 метра

Получается, что на льду длина тормозного пути практически в семь раз выше, относительно сухого асфальта (так же как и подставляемый коэффициент). На длину тормозного пути влияет качество зимней резины, физические свойства.

Тестирование показало, что с системой АБС остановочный путь существенно снижается, но все же при гололеде и снеге АБС не влияет, а наоборот ухудшает эффективность торможения, если ее сравнивать с тормозной системой без ABS. Тем не менее, в АБС по большей мере все зависит от настроек и наличия системы распределения тормозного усилия (ЕБД).

Преимущество АБС в зимнее время – полный контроль над управлением автомобиля, что сводит к минимуму возникновения неуправляемого заноса при выполнении торможения. Принцип работы АБС схож с выполнением ступенчатого торможения на автомобилях без АБС.

Система АБС уменьшает тормозной путь на: сухом и мокром асфальте, укатанном гравии, разметке .

На льду и укатанном снеге использование АБС увеличивает тормозной путь на 15 — 30 метров, но позволяет сохранить контроль над машиной, без увода машины в занос. Этот факт следует учитывать.

Как тормозить на мотоцикле?

Правильно тормозить на мотоцикле задача довольно сложная. Можно тормозить задним колесом, передним, либо двумя, юзом или двигателем. При неправильном торможении на больших скоростях можно потерять равновесие. Для того, чтобы рассчитать тормозной путь мотоцикла на 60 км/ч также подставляют данные в формулу. Учитывая при этом другой тормозной коэффициент и коэффициент трения.

Тормозной путь мотоциклов

  • Сухой асфальт: 23 — 33 метра
  • Мокрый асфальт: 35 — 46 метра
  • Грязь и снег: 70 — 95 метра
  • Гололед: 95 — 128 метра

Второй показатель – тормозной путь при торможении мотоцикла юзом.

Длину тормозного пути должен знать и уметь рассчитать любой владелец транспортного средства, и лучше это делать визуально.

Следует помнить, что при возникновении дорожно-транспортного происшествия по длине юза, который останется на дорожном покрытии, можно определить скорость движения транспортного средства перед столкновением с препятствием, что может констатировать превышение допустимой скорости водителем и сделать из него виновника происшествия.

Прием заказов на покупку автомобиля стартовал в марте, и уже на днях автомобиль поступил в продажу. Новинку отличает переработанный задний бампер с центральной подножкой, рулевое колесо и рычаг коробки передач с кожаной отделкой.

На индийском рынке появится доработанный кроссовер Suzuki Vitara Brezza. Автомобиль выйдет под брендом Toyota в 2019 году. Модель пользуется большим спросом на местном рынке. В общей сложности, было продано 210 тысяч единиц.

  • 10 — 11 классы
  • Физика
  • 5 баллов

Определить тормозной путь автомобиля если он двигался со скоростью 72км/ч и остановился за 5 с

«>

litezona.ru

Б.М. Тишин. К вопросу уточнённого расчёта тормозного и остановочного пути транспортного средства при анализе дорожно-транспортных происшествий и производстве автотехнических экспертиз

Б. М. Тишин,

негосударственный судебный эксперт в области автотехнической экспертизы,

кандидат технических наук

(г. Санкт-Петербург)

 

Расстояния тормозного и остановочного пути, рассчитанные имеющимися в экспертной практике методами, основаны на допущении о равенстве скорости движения транспортного средства на всём протяжении процесса торможения. В работе предложена методика уточнённого расчёта расстояний тормозного и остановочного пути транспортных средств, учитывающая снижение скорости на всех этапах процесса торможения. Рассчитанные расстояния методом уточнения дают результат на 10÷20 % меньше, чем по методикам, имеющимся в распоряжении экспертов сегодня.

 

Ключевые слова: методика расчёта; тормозной путь; остановочный путь; равенство скоростей; снижение скорости; погрешность результатов; замедление; время движения.

 

Т 47

ББК 67.52

УДК 343.983.25

ГРНТИ 10.85.31

Код ВАК 12.00.12

 

To the question of the refined calculation of the braking and stopping distance of the vehicle in the analysis of road accidents and the production of auto-technical examinations

 

B. M. Tishin,

non-state forensic expert in the field of autotechnical expertise

(city Sankt-Peterburg)

 

The distances of the braking and stopping tracks, calculated by the methods available in expert practice, are based on the assumption that the speed of the vehicle is equal throughout the braking process. In the work the technique of the refined calculation of distances of a brake and stopping way of vehicles, taking into account speed reduction at all stages of process of braking is offered. Calculated distances by the refinement method give a result of 10 ÷ 20 % less than the methods available to experts today.

 

Keywords: calculation technique; braking distances; stopping way; equality of speeds; reduction in speed; error in results; slowing down; driving time.

_____________________________________

 

Наиболее объективным показателем, по которому можно судить о скорости движения перед торможением, являются следы, оставленные шинами транспортного средства на дорожном покрытии.

Скорость движения транспортного средства перед торможением в экспертной практике рассчитывают по формуле:

Здесь:

— установившееся замедление при торможении транспортного средства;

— нормативное время нарастания замедления;

— длина замеренного следа торможения до остановки транспортного средства.

В данной формуле учитывается то обстоятельство, что при нажатии на педаль тормоза происходит постепенное нарастание замедления, и поэтому в формуле учитывается изменение скорости за время нарастания замедления как средняя величина при начальном замедлении «0» и конечном – «».

Однако изменение скорости движения в процессе торможения происходит не только за время нарастания замедления, но и за время срабатывания тормозного привода и за время движения транспортного средства, когда водитель принимает решение  о необходимости торможения, прекращает подачу топлива и переносит ногу с педали подачи топлива на педаль тормоза. В это время транспортное средство двигается под действием силы инерции, преодолевая сопротивление движению транспортного средства в зависимости от условий движения и сопротивление принудительному прокручиванию коленчатого вала двигателя от колёс через трансмиссию, если не выключена передача на коробке переключения передач (КПП), так как обороты коленчатого вала резко уменьшаются после прекращения подачи топлива, а колёса продолжают вращение какое-то время, практически, с прежней скоростью.

В настоящее время наличие в системе тормозов устройства антиблокировки колёс (АБС), не позволяет колёсам блокироваться при интенсивном (экстренном) торможении. Поэтому следов торможения, как таковых, на дорожном покрытии не остаётся. Это положение закреплено в ГОСТ Р 51709–2001 п. 4.1.16: «АТС, оборудованные антиблокировочными тормозными системами (АБС), при торможениях в снаряжённом состоянии, (с учётом массы водителя), с начальной скоростью, не менее 40 км/час, должны двигаться в пределах коридора движения без видимых следов увода и заноса, а их колёса не должны оставлять следов юза на дорожном покрытии до момента отключения АБС при достижении скорости движения, соответствующей порогу отключения АБС (не более 15 км/час). Функционирование сигнализаторов АБС должно соответствовать её исправному состоянию».

Это же обстоятельство не позволяет устанавливать скорость транспортного средства перед торможением по приведённой формуле, учитывающей изменение скорости за время нарастания замедления.

Поэтому скорость движения перед торможением устанавливается следствием, судом, экспертами другими методами, когда и изменение скорости за время нарастания замедления не учитывается.

Согласно ГОСТ Р 51709–2001[1], под тормозным путём понимается расстояние, пройденное АТС от начала до конца торможения.

Тормозная диаграмма, приведённая в ГОСТ Р 51709–2001 в приложении «Б» изображена на рис. 1.

Рис. 1. Тормозная диаграмма: время запаздывания тормозной системы;  время нарастания замедления;  время торможения с установившимся замедлением;  время срабатывания тормозной системы;  установившееся замедление АТС; Н и К – начало и конец торможения соответственно.

 

Начало торможения – это момент времени, в который транспортное средство получает сигнал о необходимости осуществить торможение. Обозначено точкой «Н» в приложении «Б».

Конец торможения – это момент времени, в который исчезло искусственное сопротивление движению АТС или оно остановилось. Обозначено точкой «К» в приложении «Б».

В приложении «Г» (ГОСТ Р 51709–2001) указано, что допускается вычисление тормозного пути  в метрах, для начальной скорости торможения   по результатам проверок показателей замедления АТС при торможении по формуле (приложение «Д»):

где: — начальная скорость торможения АТС, км/час;

 – время запаздывания тормозной системы, с;

— время нарастания замедления, с;

— установившееся замедление, м/с2;

В приложении «Д» первое слагаемое выражения тормозного пути приравнивается к выражению, в котором «А» – коэффициент, характеризующий время срабатывания тормозной системы.

В этом же приложении даётся таблица значений коэффициента «А», и  нормативного установившегося замедления для различных категорий АТС.

Данный способ расчёта применяется при пересчётах нормативов тормозного пути.

Таблица Д. 1

 

АТС

Категория АТС (тягач в составе автопоезда)

Исходные данные для расчета норматива тормозного пути  АТС в снаряженном состоянии:

А

 м/с2

Пассажирские и грузопассажирские автомобили

М1

0,10

5,8

М2, М3

0,10

5,0

Легковые автомобили с прицепом прицприприцепом

M1

0,10

5,8

Грузовые автомобили

N1, N2, N3

0,15

5,0

Грузовые автомобили с прицепом (полуприцепом)

N1, N2, N3

0,18

5,0

Исходя из нормативных значений коэффициента «А», для АТС категорий М1, М2, М3, расстояние тормозного пути увеличивается на 10 % от величины начальной скорости. Для АТС категорий N1, N2, N3 без прицепа – на 15 % от величины начальной скорости. Для АТС категорий N1; N2; N3 с прицепом или полуприцепом – на 18 % величины начальной скорости.

Начальная скорость подставляется в км/час.

В практике анализа ДТП или при производстве автотехнических экспертиз для определения эффективности торможения принимается не тормозной путь, обусловленный техническими параметрами автотранспортного средства, а остановочный путь АТС, обусловленный как техническими параметрами транспортного средства, так и психофизиологическими возможностями водителя.

По определению, данному профессором С. А. Евтюковым [1] – остановочный путь – это расстояние, необходимое водителю для остановки транспортного средства с помощью торможения при начальной скорости торможения при движении в конкретных дорожных условиях. Остановочный путь складывается из расстояния, проходимого транспортным средством за время реакции водителя на опасность, запаздывания тормозного привода и нарастания замедления при экстренном торможении, а также расстояния, проходимого транспортным средством с установившемся замедлением вплоть до полной его остановки.

Как видно из определений тормозного и остановочного пути, они отличаются друг от друга на расстояние, которое проходит транспортное средство за время реакции усреднённого водителя.

В экспертной практике остановочный путь рассчитывается, исходя из нормативов времени реакции усреднённого водителя, по видам дорожно-транспортных ситуаций, нормативного времени запаздывания тормозного привода и нарастания замедления по категориям транспортных средств и видам тормозных приводов.

где: — время реакции водителя, выбираемое экспертом по таблицам дифференцированных значений времени реакции водителя, в соответствии с метеорологическими и дорожными условиями [2].

— нормативно-технические значения параметров торможения, принимаемые экспертом по таблицам экспериментально расчётных значений параметров торможения автотранспортных средств в экспертной практике [3].

Как для расчёта тормозного пути по формуле, приведённой в ГОСТ, так и для расчёта остановочного пути по формуле, применяемой в практике экспертных расчётов, сделаны допущения: начальная скорость движения транспортного средства перед торможением принимается равной скорости и при нажатии на педаль тормоза и при начале движения в заторможенном состоянии с установившемся замедлением. То есть условно принимается, что на всём протяжении процесса торможения до момента возникновения установившегося замедления, скорость движения транспортного средства остаётся постоянной.

На самом деле, в процессе торможения постоянно происходит снижение скорости как при движении за время реакции водителя, так и при движении за время срабатывания тормозной системы. При расчёте тормозного и остановочного пути в приведённых формулах применяются параметры, учитывающие расстояния, которые проходит транспортное средство на этапах торможения, но не учитывается, что эти расстояния транспортное средство проходит с постоянно уменьшающейся скоростью.

При движении транспортного средства во время реакции водителя оно под действием силы инерции проходит расстояние , преодолевая силу сопротивления качению по фактическому дорожному покрытию, и, если при нажатии на педаль тормоза не происходит выключения передачи КПП, то и преодолевая силу сопротивления движению от прокручивания коленчатого вала двигателя через трансмиссию.

Сила сопротивления качению транспортного средства в общем случае определяется произведением коэффициента сопротивления качению на фактическом покрытии дороги на силу тяжести транспортного средства:

При движении на горизонтальном участке пути или когда уклоном – подъёмом можно пренебречь,

Сопротивление движению транспортного средства, возникающее от  прокручивания коленчатого вала двигателя, очень сложно рассчитать аналитически, поэтому в практике теории движения автомобилей силу сопротивления движению, возникающую от прокручивания вала двигателя через трансмиссию, рассчитывают по эмпирической формуле Ю. А. Кременца [4]:

где — рабочий объём двигателя (литраж), в литрах;

— скорость движения транспортного средства перед торможением в км/час.

— сила тяжести транспортного средства, кг.

Если движение осуществляется не на прямой передаче, то в числитель вводится передаточное число КПП передачи.

Сложность учёта этих параметров заключается в том, что для каждого конкретного случая необходимо вычислять свои значения замедления, возникающего при  преодолении сопротивлений движению. Однако это же и повышает точность произведённых расчётов остановочного и тормозного пути.

Замедление транспортного средства при преодолении сопротивления движению определяется по общей формуле замедления:

где — суммарное значение коэффициента сопротивления движению.

В частности, оно включает в себя коэффициент сопротивления качению  и условный коэффициент сопротивления от прокручивания вала двигателя через трансмиссию – .

Коэффициент  рассчитывается по общей формуле – сила сопротивления, поделённая на силу тяжести транспортного средства.

Замедление транспортного средства, возникающее при движении за время реакции водителя:

За время реакции водителя происходит снижение скорости движения:

 м/c

В момент начала реагирования на опасность скорость движения транспортного средства ,  а в момент нажатия на педаль тормоза –

                          м/с

Следовательно, всё время движения транспортного средства за время реакции водителя следует рассматривать, как движение со средней скоростью:

Исходя из представленного расчёта, к моменту начала срабатывания тормозной системы скорость транспортного средства будет не

м/с

При движении транспортного средства за время срабатывания тормозной системы (, конец движения осуществляется со скоростью:

                                        м/с

Движение транспортного средства за время срабатывания тормозной системы осуществляется со средней скоростью:

Снижение скорости за время срабатывания тормозной системы

                       

Таким образом, к моменту появления установившегося замедления скорость транспортного средства равна

Именно эту скорость следует подставлять в слагаемое, определяющее расстояние перемещения транспортного средства за время движения с установившимся замедлением до остановки или до заданного значения.

 Предложенная методика учёта снижения скорости позволяет предложить другой вариант расчёта остановочного и тормозного пути:

Несмотря на громоздкость предложенных выражений, они несложны в вычислениях, так как здесь приведены общие выводы. При последовательном решении значений средних скоростей по начальным и конечным скоростям, процесс вычислений упрощается.

Рассмотрим какое-либо конкретное событие торможения легкового транспортного средства категории , при времени реакции водителя на опасность, равном 1 с, времени запаздывания тормозного привода равным 0,1 с, времени нарастания замедления, возникающего на сухом асфальтовом покрытии 0,35 с, при установившемся замедлении 6,8 м/с2. Рабочий объём двигателя 2 л, фактическая масса транспортного средства 1500 кг, начальная скорость движения транспортного средства перед торможением 90 км/час (25 м/с). Установившееся замедление принято без учёта влияния системы АБС.

Замедление в процессе движения транспортного средства за время реакции равно:

 м/с2

где — коэффициент сопротивления качению на сухом горизонтальном асфальте – 0,018 [1].

— условный коэффициент сопротивления прокручиванию коленчатого вала двигателя через трансмиссию:

Замедление транспортного средства за время реакции водителя:

При движении за время реакции водителя происходит снижение скорости движения:

Средняя скорость движения за время реакции водителя:

Скорость в конце времени реакции:

Установившееся замедление за время срабатывания тормозной системы:

Снижение скорости за время срабатывания тормозной системы:

 

Средняя скорость движения за время срабатывания тормозной системы.

Скорость движения в конце времени срабатывания тормозной системы:

Именно эта скорость и должна подставляться в слагаемое, определяющее расстояние движение транспортного средства в режиме торможения с установившимся замедлением.

Рассчитаем расстояние тормозного пути по формулам, принимаемым в ГОСТ и по предложенной методике:

По методике ГОСТ Р 51709–2001, приложение «Д»:

По методике, допускаемой приложением «Г», ГОСТ Р 51709–2001:

По предложенной методике уточнённого расчёта:

Что составляет, соответственно, 19,8 и 16,6 % от величины тормозного пути, определённого по ГОСТ Р 51709–2001.

По принятой в экспертной практике методике расчёта расстояния остановочного пути:

По предложенной методике уточнённого расчёта:

Что составляет 11,6 % от величины тормозного пути, рассчитанного  по принятой методике:

Предлагаемая методика позволяет учитывать влияние конкретной модели транспортного средства и при дифференцированном расчёте тормозного и остановочного пути уменьшить погрешность расчёта. Это позволяет принимать категорический вывод о наличии или отсутствии технической возможности предотвращений дорожно-транспортных происшествий на более обоснованных расчётах, а не на усреднённых нормативных параметрах и допущении о равенстве скорости движения в процессе всего процесса торможения до момента возникновения установившегося замедления.

Применяемые в экспертной практике формулы расчёта тормозного и остановочного пути дают завышенный результат, превышающий 10 %, по сравнению с предлагаемой методикой уточнённого расчёта. При расчёте тормозных и остановочных путей транспортных средств категорий N1, N2, N3 по предлагаемой методике разность результатов по сравнению с применяемыми методиками будет увеличиваться, так как растёт значение коэффициента «А».

 

Литература:

1. Евтюков С.А., Васильев Я. В. Экспертиза ДТП: Справочник. — СПб.: ДНК, 2006.

2. Применение дифференцированных значений времени реакции водителя в экспертной практике: Методические рекомендации ВНИИСЭ. – М., 1987.

3. Использование в экспертной практике экстремально-расчетных значений параметров торможения АТС: Методические рекомендации ВНИИСЭ. – М., 1986.

4. Боровский Б. Е. Безопасность движения автомобильного транспорта. – Л.: Лениздат, 1984.

 

 


[1] ГОСТ Р 51709–2001. Автотранспортные средства. Требования безопасности к техническому состоянию и методы проверки.


Определение длины тормозного пути — Энциклопедия по машиностроению XXL

Определение длины тормозного пути по заданным o, v , v . Искомую длину находят, пользуясь формулой (50), которая в данном случае принимает вид  [c.234]

Расчетные нормативы нажатия тормозных колодок на ось установлены из условия получения тормозных сил при экстренном торможении. Для определения длины тормозного пути пассажирских и грузовых поездов в зависимости от скорости движения, тормозного нажатия и величины спусков необходимо руководствоваться номограммами, приведенными в Правилах тяговых расчетов для поездной работы, утвержденных МПС (см. приложение П1), или тормозной путь определяется при экстренном торможении аналитическим путем.  [c.70]


Все механизмы грузоподъемных машин снабжаются надежными тормозными устройствами, обеспечивающими в механизмах подъема- остановку груза и удержание его в подвешенном состоянии, а в механизмах передвижения и поворота — остановку механизма на определенной длине тормозного пути.  [c.155]

Иногда в эксплуатационной практике могут возникнуть и такие задачи, как определение длины тормозного пути или величины уклона. Тогда заданными будут скорость и тормозные средства.  [c.35]

Механизмы грузоподъемных машин должны быть снабжены падежными тормозными устройствами, обеспечивающими в механизмах подъема остановку груза и удерживание его в подвешенном состоянии, а в механизмах передвижения и поворота остановку механизма на определенной длине тормозного пути с последующим удерживанием механизма в неподвижном состоянии.  [c.96]

В гл. VI исследуется процесс торможения пневматических устройств в конце хода поршня. Рассматривается приближенный >4. метод определения длины тормозного пути, а также других пара- s метров торможения. Приводятся результаты испытаний.  [c.17]

Для определения длины тормозного пути необходимо знать, как будет меняться закон падения скорости поршня при торможении в зависимости от момента включения тормозных устройств, т. е. от Х . С этой целью на рис. 105, а построены графики X = = X (X) для различных значений и при т] = 0,1. Кривые скорости располагаются почти эквидистантно, и тормозной путь Х остается практически тем же самым.  [c.258]

Локомотивная бригада должна видеть каждый сигнал на расстоянии не менее служебного тормозного пути, на котором она может реализовать торможение для остановки поезда, если на светофоре останется красный огонь. Численное значение определяют тормозные средства поезда, скорость его следования и профиль пути. Взаимосвязь этих элементов находят специальными расчетами и проверяют опытными поездками, на основании которых составляют таблицу длин расчетного тормозного пути при экстренном пневматическом торможении — ответственный документ безопасности движения поездов. В этой таблице даны расчетные тормозные пути при экстренном автостопном торможении, поэтому для определения длины тормозного пути при служебном электрическом торможении длину пути, указанную в таблице, следует умножить на коэффициент 1,15. Например, при скорости 70 км/ч (19,4 м/с) тормозной путь при автостопном (экстренном) торможении для состава из семи вагонов типа Д на  [c.33]


Определение длины тормозного пути  [c.464]

Определение длины тормозного пути 461, 462  [c.490]

Процесс торможения. Механизмы поворота грузоподъемных машин в машинным приводом имеют тормоза, обеспечивающие остановку механизма на определенной длине тормозного пути. Тормоз должен останавливать поворотную часть крана на допустимом пути торможения при действии ветра в направлении поворота и допустимого уклона пути для рабочего состояния крана и обеспечивать плавное торможение при отсутствии ветра. На механизмах поворота с ручным приводом установка тормозов не обязательна, если исключена возможность самопроизвольного поворота под действием ветровой или какой-либо иной нагрузки.  [c.339]

Стендовые испытания тормозных накладок проводят путем разгона инерционных масс до определенной скорости, соответствующей скорости движущегося автомобиля, после чего приводят в действие тормоз. Кинетическая энергия вращающихся масс преобразуется в тормозе главным образом в тепловую энергию, в результате чего температура накладок повышается. Проводят серию таких торможений с интервалами 30—40 с до тех пор, пока температура не достигнет установленного уровня. Определяют зависимость тормозного пути от температуры. По длине тормозного пути  [c.219]

Показания входных сигналов днем и ночью должны быть отчетливо видны с приближающегося поезда на расстоянии не менее длины тормозного пути, определенного для данного места пути при полном служебном торможении и максимальной реализуемой скорости, но не менее 1000 м. Если по местным условиям нельзя обеспечить хорошую видимость показаний входных светофоров и семафоров на указанном расстоянии, перед ними устанавливают предупредительные светофоры. Эти светофоры применяют также в районах с частыми туманами, метелями и на участках с интенсивным движением поездов. От входных светофоров или семафоров предупредительные светофоры устанавливают на расстоянии не менее длины тормозного пути, определенной для данного места при экстренном торможении и максимальной реализуемой скорости, но не менее 1000 м на линиях, оборудованных автоблокировкой, нет предупредительных светофоров, так как каждый проходной светофор является предупредительным по отношению к следующему сигналу. Показания входных сигналов, перед которыми установлены предупредительные, а также показания самих предупредительных сигналов должны быть отчетливо различимы на расстоянии не менее 200 м. На рис. 54 показаны расстановка входного, выходных и предупредительного светофоров на промежуточной станции, расположенной на двухпутном участке, оборудованном автоблокировкой, и условные обозначения цвета сигнальных огней.  [c.98]

Тягово-энергетические испытания — имеют целью составление тягово-энергетических характеристик и проверку соответствия последних техническим условиям на постройку данного локомотива. Эти испытания, как правило, производятся на экспериментальном кольце ЦНИИ МПС. Только определение удельного сопротивления и длины тормозных путей одиночно следующего локомотива производится на эксплуатируемых участках железнодорожных линий.  [c.203]

Определение допускаемой скорости движения. При заданной длине тормозного пути и тормозных средств поезда допустимую скорость определяют графическим способом. На оси абсцисс откладывают тормозной путь, а затем с помощью кривых замедляющих сил + + = f ( ) строят кривые v (s) для различных уклонов (рис. 210) 320  [c.320]

Контрольно-осмотровые работы, основное назначение которых — предупреждение и обнаружение неисправностей в механизмах автопогрузчика, могущих повлечь за собой поломки и аварии его в процессе работы, а также выявление несоответствия состояния отдельных агрегатов и автопогрузчика в целом действующим техническим требованиям и условиям, проводят путем внешнего осмотра, опробования в действии отдельных узлов и автопогрузчика в целом, замера расстояний и зазоров между деталями и снятия некоторых характеристик (например, определенные усилия на рулевом колесе, длина тормозного пути).  [c.158]

Приведенное аналитическое выражение для определения тормозного пути соответствует пути с момента полного включения тормозов. В действительных условиях с момента возникновения необходимости торможения до полного включения тормозов проходит некоторый промежуток времени, в течение которого действительная длина тормозного пути увеличивается.  [c.720]

При выборе длины тормозного пути нужно также учитывать величину противодавления, оно не должно превышать определенного предела. Однако во всех рассмотренных случаях величина  [c.261]

К задачам оптимального стратегического управления можно отнести выбор видов тяги и постройку локомотивов с необходимыми тяговыми характеристиками и вагонов с определенными параметрами выбор оптимальных расчетных скоростей и силы тяги локомотивов установление наибольших допускаемых скоростей движения поездов и длины тормозного пути установление среднесетевого уровня участковой скорости, веса поезда, расхода энергоресурсов и др. Конкретное разрешение стратегических задач управления находит отражение в графике движения поездов, безусловное выполнение которого составляет главную задачу эксплуатации тяговых средств.  [c.263]

Длина каждого блок-участка должна быть не менее длины тормозного пути поезда при полном служебном торможении (при использовании 0,8 полной тормозной СИЛЫ). Выполнение этого условия проверяется решением на каждом блок-участке тормозной задачи на определение полного тормозного пути 5т при известном значении начальной скорости У , как это показано на рис. 22-1X.  [c.142]

Светофоры на перегонах при автоблокировке имеют двух-, трех- и четырехзначные показания. Они должны быть отчетливо видимы с приближающегося поезда на расстоянии не менее расчетного тормозного пути при полном служебном торможении и максимальной скорости движения на данном участке. Расстояние между двумя смежными светофорами на перегонах определяют специальными (тяговыми) расчетами, учитывая безопасность движения и заданную пропускную способность линии. Оно должно быть не менее расчетной длины тормозного пути, определенного для данного места, при полном служебном торможении и максимальной реализуемой скорости движения поездов между светофорами.  [c.18]

Для автостопного торможения тормозной путь должен быть менее расстояния между постоянными сигналами. Длину тормозного пути в этом случае определяют, так же как при экстренном торможении. Разница состоит лишь в том, что при определении подготовительного тормозного пути время подготовки тормозов к действию увеличивают на 14 с.  [c.49]

Уточненное определение тормозного пути с учетом раскачивания груза при большой длине подъемных канатов см. [12].  [c.386]

По условиям безопасности движения любой поезд должен быть остановлен на расстоянии, равном длине расчетного (полного) тормозного пути. Однако с увеличением крутизны спусков затрудняются условия остановки поезда торможением, и поэтому для соблюдения заданной величины расчетного тормозного пути при переходе на спуски большей крутизны необходимо, следовательно, или увеличивать число тормозных вагонов в поезде, или уменьшать скорости движения. Отсюда и возникает необходимость определения на спусках различной крутизны наибольших допускаемых скоростей движения при заданной длине расчетного тормозного пути и при заданных тормозных средствах,  [c.187]

Это обстоятельство требует установления определённой зависимости между скоростью движения поезда V, уклоном профиля участка пути г, наличием тормозных средств (количеством тормозных осей) или тормозным коэффициентом и длиной расчётного тормозного пути 5 . Эта зависимость определяется тормозными расчётами, которые связывают четыре указанные величины, и решение тормозных задач сводится к определению одной из этих величин при остальных заданных.  [c.35]

На конвейере с перемещением вниз тормоз должен не только удерживать конвейер от самопроизвольного движения, но и остановить движущийся конвейер, т. е. в течение определенного промежутка времени (на определенной длине пути) поглотить кинетическую энергию груза и движущихся частей конвейера. В обычных условиях это можно учесть, приняв несколько повышенную величину тормозного момента,  [c.103]

Обычно задаются длиной пути 5, на котором должен быть заторможен конвейер. При этом для определения потребного тормозного момента Му на валу двигателя можно воспользоваться уравнением (3. 128), в котором вместо нужно подставить его значение из уравнений (3. 132) и (3. 133), а вместо е — его значение из уравнения (3. 134 )  [c.104]

Обычно задаются длиной пути 5, на котором должен быть заторможен конвейер. При этом для определения потребного тормозного момента М- на валу двигателя можно воспользоваться уравнением (3.73), в котором вместо Мст  [c.97]

На участках, где автоматическая локомотивная сигнализация применяется как самостоятельное средство сигнализации и связи, длина двух смежных блок-участков должна быть не менее тормозного пути, определенного для данного места при экстренном торможении с учетом пути, проходимого поездом за время, необходимое для воздействия устройств автоматической локомотивной сигнализации и автостопа на тормозную систему поезда при максимальной реализуемой скорости.  [c.39]

Графический метод определения тормозного пути основан на графическом решении уравнения движения поезда и аналогичен графическому методу определения скорости и времени хода поезда по перегонам. В этом случае строится кривая удельных замедляющих сил в зависимости от скорости движения поезда (см. рис. 9 и 10). Путем несложных Построений, пользуясь этой кривой и переместив начало координат на отрезок, соответствующий установленному в условии задачи уклону, по которому происходит движение поезда, строят кривую зависимости скорости движения от пройденного пути у(5) (если поезд движется по площадке, то переменить начало координат не требуется). По кривой падения скорости поезда вследствие действия тормозных сил можно определить длину действительного пути 5д для различных значений начальной Уд и конечной скоростей движения.  [c.49]

При формировании поездов из групп вагонов, находящихся на разных путях, их переставляют и объединяют согласно плану работы по разрешающим показаниям маневровых светофоров и указаниям составителя. Выезжая с пути, машинист обращает внимание на длину переставляемой группы вагонов и ее ориентировочную массу. Массу необходимо учитывать для определения тормозного пути, снижения скорости и места остановки, длину — при определении расстояния свободного участка пути перед стоящей группой вагонов.  [c.255]

Процесс торможения. Механизмы поворота кранов, работающих на открытом воздухе, а также кранов, работающих в помещении (группы классификгщии режима работы М2 и более), должны быть оборудованы тормозами, обеспечивающими прекращение движения на определенной длине тормозного пути. Тормозной путь не должен превышать допустимого значения при действии ветра рабочего состояния в направлении поворота при допустимом уклоне пути. При отсутствии ветра тормозное устройство должно обеспечивать плавное торможение. При работе на открытом воздухе тормоз должен обеспечивать остановку поворотной части крана при действии максимально допустимой скорости ветра (по ГОСТ 1451 — 77) для рабочего состояния крана с учетом допустимого уклона.  [c.459]

Видимость и расстановка сигналов. В соответствии с требованиями Правил технической эксплуатации показания входных и проходных сигналов, не имеющих предупредительных светофоров или дисков, должны быть днем и ночью отчетливо различимы с приближающегося поезда на расстоянии не менее тормозного пути, определенного для данного места при полном служебном торможении и максимальной реализуемой скорости, но не менее 1000 м. Видимость предупредительных сигналов, а также основных сигналов, перед которыми установлены предупредительные сигналы, должна быть не менее 400 м, в сильно пересеченной. честности (горы, глубокие выемки) допускается видимость менее 400 м, но не менее 200 м. Если предупредительный сигнал виден за 400 м и более, то, будучи установленным от основного сигнала на расстоянии длины тормозного пути, он обеспечит остановку поезда перед закрытым основным сигналом при примененип машинистом служебного торможения. Длина тормозного пути при служебном торможснин примерно Б 1,25 раза больше, чем прн зкстрекком торможении. Исходя из этого и учитывая все возрастающие размеры и скорости движения, необходимо, очевидно, в подавляющем большинстве случаев иметь предупредительные сигналы перед входными и проходными сигналами.  [c.342]

Места препятствий ограждают, учитывая максимальную длину тормозного пути для обращающихся на участке типов поездов. Управляемость тормоза подвижного состава оценивается точностью остановки поезда и снижением скорости до определенного уровня в месте ее ограничения в минимально возможное время. Наиболее высокая управляемость у электроп-невматических тормозов. В длинносоставных грузовых поездах, чтобы управляемость была высокой, автотормоза устанавливают на равнинный  [c.211]

В больишнстве случаев воздух в атмосферу выбрасывается только до определенной длины тормозной магистрали. При установленном режиме отпуска тормозов выброс воздуха в атмосферу в пассажирских поездах происходит при длине тормозной магистрали до 12 четырехосных вагонов и в грузовых поездах до 20 четырехосных вагонов, а при большем числе вагонов, как правило, выброс (сброс) не происходит. Этим свойством машинисты локомотивов могут пользоваться для контроля перекрытия концевых кранов, образования ледяных пробок тормозной сети поезда при торможении в пути следования.  [c.151]

Как видим, процесс торможения поезда определяется четырьмя указанными нормативами. Решение тормозных задач сводится к нахождению одного из них при известных трех других графическим либо аналитическим методом. Условно тормозные задачи делятся на две основные группы. В первой из них определяют допускаемую скорость движения при заданном тормозном пути, известных тормозных средствах и профиле пути, либо находят тррмозной путь в зависимости от заданной максимальной (начальной) скорости движения, силы нажатия тормозных колодок и профиля пути. Ко второй группе относятся задачи по определению необходимой силы нажатия тормозных колодок при заданных максимальной допустимой скорости движения, длине тормозного пути и уклоне.  [c.48]

С этой целью по данным диаграммы удельных замедляющих сил строят кривую зависимости их от скорости. Задаются несколькими произвольными значениями спусков, например О, — 4, — 8, — 12 %о. Для каждого из них по диаграмме удельных замедляющих сил, перенося начало координат в точки, соответствуюхцие принятым спускам, строят кривые зависимости скорости от пройденного пути у(з). Принимая линейной зависимость тормозного пути от начальной скорости, наносят на график ф) кривые зависимости 5д уд) таким образом, чтобы они пересекли ось абсцисс в точках, соответствующих длине тормозного пути 1000 и 1200 м. Тогда точки пересечения кривых v(s) и 5д(у ) будут определять допускаемые скорости для соответствующих спусков. После этого наносят соответствующие точки на планшет и, соединив их плавной кривой, получают зависимость для тормозных путей 1000 и 1200 м. Используя построенный таким образом график, можно решать следующую задачу определение наибольшей допускаемой скорости по тормозам для спуска любой крутизны.  [c.50]

Уравнение (2) может быть использовано для определения длины условного тормозного пути х т в зависимости -от значений установившейся скорости, нагрузки и вреднего пространства. В работе [5] приведен график для определения, хг при полном падении скорости. Там же приведены формулы для вычисления времени тор-мол[c.222]

Выведенные формулы не учитывают влияния возможного раскачивания груза при торможении и являются полностью справедливыми для таких кранов и тележек, с которыми груз жестко связан (например, для клещевых кранов и штабелеров). Как показывают исследования, влияние раскачивания груза на движение крана или тележки зависит главным образом от соотношения времени их разгона и периода качания груза на по-лиспастной подвеске и от соотношения между массой груза и массой крана или тележки. За время торможения большинства механизмов передвижения груз не успевает совершить полного колебания около положения равновесия. Поэтому для подавляющего большинства конструкций механизмов передвижения определение значения замедления и длины пути торможения по приведенным выше формулам обеспечивает достаточную точность расчета. Уточненное определение тормозного пути с учетом раскачивания груза приведено в [10, 14].  [c.402]

В заключение производится настройка основной пружины /2 на заданный прогиб, так как установочная длина пружины при замкнутых колодках должна соответствовать определенному значению тормозного момента по паспорту тормоза. Длина пружины регулируется вращением в соответствующую сторону п.гтока 8 за его квадратный хвостовик. При этом гайка // должна удерживаться от проворачивания. Если установочное значение длины пружины неизвестно, то регулировку тормоза можно производить по выбегу механизма под нагрузкой, т. е. по тормозному пути от начала торможения до полной остановки.  [c.51]

Кроме того, машинисты всех видов движения иногда запаздывают при выборе момента начала торможения перед запрещающим сигналом. В результате этого, как показьшает анализ, поезд проходит путь длиной до 300-400 м и более (когда машинист максимально собран и концентрирует внимание на опасности). Конечно, имеются и другие причины ошибок, но наибольшее значение и здесь имеет время реакции. Почему так Прежде всего потому, что машинист работает на постоянных (одном или нескольких) участках обращения, производит сотни торможений с разными поездами и в различных погодных условиях и накапливает определенные навыки и умения в прогнозировании необходимого тормозного пути, скорости и тормозной эффективности. Кроме того, на это направлена вся техническая подготовка на курсах и технических занятиях в депо, т. е. все другие причины можно свести к минимуму, а время реакции трудно снизить.  [c.104]


Определение длины тормозного пути — Страница 2

Страница 2 из 2

Тормозной путь определяют исходя из скорости движения, расчетного тормозного нажатия н профиля пути С помощью расчетных номограмм тормозного пч’ти при экстренном торможении определяют одно из четырех условий процесса торможения при заданных трех основных (тормозной путь, максимальная начальная скорость торможения, коэффициент расчетного тормозного нажатия, уклон). При расчете тормозного пути полного служебного торможения удельную тормозную силу уменьшают на 20%.
Таблица 247 Формулы для расчета длины тормозных путей и величины замедления поезда


Примечания I Номограммы величины тормозного пути в зависимости от расчетного тормозного коэффициента и скорости в начале торможения приведены для грузовых поездов на рис 315 и 316 и для пассажирских — на рис. 317 и 318.

  1. Номограмма величины тормозного пути в зависимости от скорости движения и среднее замедления поезда приведена на рис. 319.
  2. Величина среднего замедления представляет собой удельную кинетическую энергию, приходящуюся на единицу массы, которая гасится тормозной системой на единице длины тормозною пути:

для пассажирских и моторвагонных поездов на площадке  для грузовых и пассажирских поездов

где ?τ — время от начала торможения до полной остановки поезда

  1. Расчетный коэффициент сцепления колес с рельсами определяют по формуле


где о — средняя нагрузка от колесной пары на рельсы Значение функции скорости см на рис 320
Таблица 248. Величина замедления ς, км/ч2 под действием удельной замедляющей силы 1 кгс/т


Подвижной состав

Замедление

Грузовые и пассажирские поезда Одиночно следующие локомотивы:

120

паровозы

121

тепловозы

114

электровозы

107

Электропоезда

119

Дизель-поезда

116


Рис. 315. Номограмма величины тормозного пути грузового поезда при чугунных колодках:
а — на площадке, б -на спуске 0,006, в — на спуске 0,010

Рис. 316. Номограмма величины тормозного пути грузового поезда при композиционных тормозных колодках: а — на площадке; б — на спуске 0,006; в — на спуске 0,010

Рис. 317. Номограмма величины тормозного пути пассажирского поезда при чугунных тормозных колодках (сплошные линии — электропневматическое торможение, штриховые — пневматическое):
а — на площадке, б — на спуске 0,006, в — на спуске 0,010

Рис. 318. Номограмма величины тормозного пути пассажирского поезда при композиционных тормозных колодках (сплошные линии — электропневматическое торможение, штриховые — пневматическое):  а — на площадке; б — на спуске 0,006, в — на спуске 0,010


Рис. 319. Номограмма величины тормозного пути в зависимости от скорости и замедления поезда на площадке

Таблица 249. Формулы для определения времени подготовки тормозов к действию


Тип поезда

Время подготовки tп, с

Грузовой состав длиной до 200 осей при пневматических тормозах


1 При срабатывании автостопа время подготовки тормозов к действию увеличивается на 12 с
Таблица 250. Формулы для определения коэффициентов трения тормозных колодок о колесо

Таблица 251. Расчетный коэффициент трения тормозной колодки о колесо

Примечание Действительный коэффициент трения тормозной колодки о колесо определяется по формулам:  где К — действительная сила нажатия тормозной колодки на колесо, тс.

Таблица 252. Расчетная сила нажатия тормозной колодки на колесо Кр в зависимости от действительной силы нажатия К

Примечание. Действительная сила нажатия тормозной колодки на колесо определяется по формуле K = Fpm\u, кгс, где h -площадь поршня тормозного цилиндра, см2, р — давление сжатого воздуха в тормозном цилиндре, кгс/см2; п — передаточное число рычажной передачи до колодки; нп — коэффициент полезного действия рычажной передачи (с учетом влияния усилия отпускной пружины).

Т а блица 253. Расчетный коэффициент сцепления, принимаемый для проверки отсутствия заклинивания колесных пар и рекомендуемый при проектировании тормозного оборудования


Расчетная скорость, км/ч

Расчетный коэффициент сцепления при нагрузке от колесной пары на рельсы, тс

6

10

15

20

25

Пассажирские, изотермические вагоны, вагоны электро-  и дизель-поездов

 

 

 

 

 

40

0,140

0,135

0,130

0,124

__

120

0,110

0,107

0,102

0,097

140

0,106

0,102

0,098

0,094

__

160

0,101

0,097

0,094

0,090

__

Грузовые вагоны

 

 

 

 

20

0,31

0,125

0,121

0,116

0,110

100

0,097

0,094

0,090

0,086

0,081

120

0,092

0,090

0,085

0,081

0,070

Локомотивы

20

__

0,132

0,126

0,119

100

_

0,097

0,093

0,088

160

0,087

0,083

0,078

Таблица 254. Тормозной путь, м, проходимый поездом при проверке действия тормозов с начальной скорости


Крутизна
спуска

Скорость, км/ч

40

60

80

100

120

0

125/250*

220/450

330/650

400/ —

555/ —

0,002

140/300

245/500

260/750

490/ —

620/ —

0,004

150/350

270/600

400/900

545/ —

695/ —

* Здесь и далее перед чертой — для пассажирских поездов, за чертой — для грузовых.
Таблица 255. Процент расчетного тормозного нажатия от максимального при ступенях торможения и чугунных тормозных колодках в грузовом поезде


Режим включения воздухораспределителя

Величина снижения давления в тормозной магистрали, кгс/см2

0,65

0,75

0,95

Порожний

65

75

90

Средний

45

57

75

Груженый

30

50

70


Рис. 320. Функция скорости для определения расчетного коэффициента сцепления колес с рельсами:
1 — пассажирский подвижной состав и вагоны на тележках пассажирского типа; 2 — локомотивы; 3 — грузовые вагоны

Рис. 321. Перепад давления Δρ в тормозной магистрали в зависимости от ее длины (м), утечки (л/мин), приходящейся на I м длины магистрали, и зарядного давления:
1 — 6,2 кгс/см2; 2 — 5,5 кгс/см2, 3 — 4,8 кгс/см2

Рис. 322. Зависимость величины зарядного давления в тормозной магистрали грузового поезда при установленном минимальном давлении в его хвостовой части от длины магистрали и равномерно распределенных утечек величиной:
1 — 2 л/мин · м; 2 — 1,4 л/мин · м; 3 — 1 л/мин м

  1. Графический способ определения диаметра калиброванного отверстия в зависимости От объема резервуара и времени истечения из него воздуха в атмосферу через калиброванное отверстие

Таблица 256. Определение времени истечения воздуха из резервуара в атмосферу (рис. 323, 324)


Рис. 323. Номограмма № 1 для определения отношения Vff в зависимости от объема Резервуара и диаметра отверстия


Рис. 324. Номограмма № 2 для определения времени истечения воздуха из резервуара в атмосферу через круглое отверстие в зависимости от отношения Vff (см. рис. 323)

Расчет тормозного пути по интервалам скорости

Наиболее распространенным при практическом использовании является аналитический метод расчета длины тормозного пути, опирающийся на численное интегрирование уравнения движения поезда (2.4) по интервалам скорости. При этом тормозной путь 5Т для упрощения расчетов разбивается на два участка: подготовительный 5П и действительный 5Д.

Условно считается, что при прохождении поездом участка 5П тормоза не работают, а на участке 5Д они действуют с максимальным и неизменным давлением в ТЦ, которое возникает скачкообразно. Участок 5П и время 1п, за которое его проходит поезд, рассчитываются таким образом, чтобы путь 5Т, полученный указанным способом, соответствовал вычисляемому с учетом реального нарастания давления в ТЦ.

Расчет длины тормозного пути выполняется по следующей формуле где — скорость поезда перед торможением, км/ч;

Ун, Ук — начальная и конечная скорости поезда в выбранном интервале скоростей, км/ч;

£ — замедление поезда под действием единичной удельной силы, кмкН/(ч2Н) (для вагонов составляет 120, тепловозов — 114, электровозов — 107, электропоездов — 119), а при расчетах для грузовых и пассажирских поездов принимается £ = 120; Ьт — удельная тормозная сила, Н/кН;

<о0х — основное удельное сопротивление движению поезда, Н/кН; 1с — удельное сопротивление от спрямленного уклона с учетом сопротивления в кривой, %о.

Так как в (9.9) входят сложные нелинейные функции, характеризующие процесс торможения и одновременно зависящие от него, то расчет второго слагаемого ведется пошагово методом численного интегрирования. При этом в выбранном интервале скоростей (для счета вручную обычно Д V = 10 км/ч, на ПЭВМ Д V = 5 км/ч) удельные силы Ьт, (о$х, 1с условно принимаются неизменными и равными для средней скорости в данном интервале. После чего рассчитывается часть длины действительного тормозного пути 5Д. Последовательно суммируя эти части от минимальной скорости до выбранной и прибавляя к ним соответствующие значения 5П можно получить зависимость длины тормозного пути данного поезда от скорости его торможения 5Т =ЛУ). Результаты расчетов удобно записывать в форме таблицы, аналогичной табл. 9.2 (для ориентировки в ней приведены данные, соответствующие характеристикам груженого грузового поезда).

Как видно из (9.9), для расчета 5П использована формула, предполагающая равномерное движение поезда, которое возможно лишь при условии со0д. = [-у. Поэтому учет изменения скорости поезда в зависимости от уклона на этом отрезке пути сделан за счет корректировки *п. Кроме того, на это время влияют длина поезда, время наполнения ТЦ и значение Ьт Таблица 9.2

Результаты расчетов длины тормозного пути поезда весом А», кН, на спуске У, %с

V, км ч

Н кН

с

5П, м

Уф, км ч

Н кН

Н кН

Н кН

Шип Н кН

Н кН

Д5Д, м

2>я-/.и м

5„ м

80

0,097

32,9

15,8

352

75

0,100

33,6

1,64

52

1,69

282

221

801

1153

70

0,102

34,6

15,6

304

65

0,105

35,6

1,48

4,6

1,54

30,1

180

580

884

60

0,108

36,6

15,4

257

55

0,112

38,0

135

4,1

1,40

32Д

142

400

657

0,198

67,1

13,9

39

5

0227

74,6

0,%

2^

1,01

68,5

6

6

45

Обобщенная формула для расчета времени подготовки тормозов к действию имеет вид

(9.Ю)

Коэффициенты А и Р для грузовых поездов с количеством осей 200 соответственно равны 7 и 10; от 200 до 300 — 10 и 15; более 300 осей — 12 и 18; для пассажирских поездов и одиночно следующих локомотивов с пневматическими тормозами — 4 и 5; для пассажирских поездов с ЭПТ — 2 и 3. При автостопном торможении рассчитанное время 1п увеличивается на 14 с. В формулах (9.5), (9.6) значение 1с принимается для спусков со знаком минус, для подъемов со знаком плюс.

Таким образом, последовательно применяя формулы (3.12), (3.11), (3.10), (3.13) и (9.10), определяют *п. Из (9.9) находят 5П, занося в табл. 9.2 соответствующие значения рассчитанных параметров. Для расчета действительного тормозного пути в выбранном интервале скоростей определяют среднюю и для нее рассчитывают Ьт (как показано выше по формуле (3.13) и основное удельное сопротивление движению щх.

Основным сопротивление движению называют потому, что оно присутствует на подвижном составе всегда и проявляется в виде сил трения между колесами и рельсами, в буксовых узлах и набегающей воздушной среде. К дополнительному сопротивлению относятся, например, сопротивление, возникающее при подъеме, в кривом участке пути, при ветре и низкой температуре, при работе подвагонного генератора, при трогании с места и ряд других, которые могут возникать на подвижном составе в процессе его эксплуатации.

Поскольку сопротивление движению в соответствии с молеку-лярно-механической природой сил трения существенно зависит от приложенной нагрузки, то для расчетов используют его удельное значение, приходящееся на единицу веса транспортного средства. Таким образом, несмотря на то что удельное сопротивление движению, например, порожнего вагона больше, чем груженого, полное сопротивление последнего будет, конечно, выше. Это объясняется тем, что темп падения удельного сопротивления с ростом нагрузки оказывается меньше, чем скорость ее увеличения,

Сопротивление перевозке единицы груза в груженом вагоне меньше, чем сопротивление в порожнем. Значит, энергозатраты на проведение по участку загруженного и порожнего поездов одинакового веса при прочих равных условиях для последнего оказываются больше.4»й)о„ — основное удельное сопротивление движению восьми,

четырехосных и других типов вагонов, Н/кН; б8, б4, би — вес соответствующей группы вагонов, кН.

Формулы для расчета со* вагонов различных категорий на звеньевом пути приведены ниже:

— грузовые четырехосные на подшипниках скольжения и шести-осные на роликовых подшипниках в груженом состоянии

— грузовые четырехосные с роликовыми подшипниками в груженом состоянии и вагоны рефрижераторных поездов

— грузовые груженые восьмиосные на роликовых подшипниках

— пассажирские цельнометаллические на роликовых подшипниках Получив для каждого интервала скоростей величины действительных тормозных путей Д5Д и сложив их последовательно от соответствующего минимальной (остановочной) до максимальной (или требуемой для построения графика), заносят в соответствующую графу 5Д табл. 9.2. Наконец, складывая эти значения с ранее рассчитанным для данной скорости движения 5П, получают величину 5Т.

⇐ | Расчет длины тормозного пути | | Автоматические тормоза подвижного состава | | Расчет тормозного пути по интервалам времени | ⇒

Тормозной путь

Тормозной путь — это расстояние, которое автомобиль преодолевает при замедление до полной остановки. Тормозной путь зависит от нескольких переменные. Во-первых, на торможение влияет уклон (уклон) проезжей части. расстояние. Если вы идете в гору, сила тяжести помогает вам в попытках остановиться и уменьшает тормозной путь. Точно так же гравитация работает против вас, когда вы при спуске и увеличит тормозной путь.Далее сопротивление трения расстояние между проезжей частью и шинами может повлиять на тормозной путь. Если у вас есть старые шины на мокрой дороге, скорее всего, вам потребуется большее расстояние, чтобы остановиться, чем если бы у вас новая резина на сухой дороге. Последний параметр, который мы рассмотрим, — это ваш начальный скорость. Очевидно, что чем выше ваша скорость, тем дольше вы будете останавливаться, учитывая постоянное замедление.

Уравнение, используемое для расчета тормозного пути, является дочерним по отношению к более общим уравнение из классической механики.Исходное уравнение приведено ниже.

Vf2 = Vo2 + 2ad

Где:
Vf = Конечная скорость
Vo = начальная скорость
a = Скорость ускорения
d = расстояние, пройденное при ускорении

При расчете тормозного пути мы предполагаем, что конечная скорость будет равна нуль. Исходя из этого, уравнением можно манипулировать, чтобы найти расстояние пройдено при торможении.

d = -Vo2 / (2a)

Обратите внимание, что расстояние будет положительным, пока отрицательная скорость ускорения использовал.

Ускорение тормозящего транспортного средства зависит от сопротивления трения и класс дороги. Из наших знаний о силе трения мы знаем, что ускорение из-за трения можно рассчитать, умножив коэффициент трения ускорением свободного падения. Точно так же мы знаем из задач наклонной плоскости что часть веса автомобиля будет действовать в направлении, параллельном поверхности Дорога.Ускорение свободного падения, умноженное на уклон дороги, даст нам оценить ускорение, вызванное уклоном дороги.

Окончательная формула тормозного пути приведена ниже. Обратите внимание, как Скорость ускорения рассчитывается путем умножения ускорения свободного падения на сумму коэффициента трения и уклона дороги.

d = V2 / (2g (f + G))

Где:
d = тормозной путь (фут)
g = ускорение свободного падения (32.2 фут / сек2)
G = уклон проезжей части в процентах; для 2% используйте 0,02
V = Начальная скорость автомобиля (фут / сек)
f = коэффициент трения между шинами и дорожным полотном

Тормозной путь и время реакции тормоза являются важными составляющими расчет дальности остановки прицела. Для того, чтобы стопорный прицел при условии достаточного расстояния, нам необходимо более глубокое понимание фрикционных сила.Значение коэффициента трения сложно определить. определять. Сила трения между шинами и дорожным полотном сильно различается. и зависит от давления в шинах, состава шин и типа протектора. Фрикционный сила также зависит от состояния поверхности дорожного покрытия. Наличие влага, грязь, снег или лед могут значительно уменьшить тормозящую силу трения ты. Кроме того, коэффициент трения ниже на более высоких скоростях.С коэффициент трения для мокрого покрытия ниже, чем коэффициент трения для сухое покрытие, мокрое покрытие используется на расстоянии видимости остановки расчеты. Это обеспечивает разумный запас прочности, независимо от состояние дорожного покрытия. В таблице ниже приведены несколько значений фрикционного коэффициент в условиях мокрого дорожного покрытия (ААШТО, 1984).

Тормозной путь, путь реакции и тормозной путь

Расстояние реакции

Дистанция реакции — это расстояние, которое вы пройдете от точки обнаружения опасности до начала торможения или поворота.

На дистанцию ​​реакции влияет

  • Скорость автомобиля (пропорциональное увеличение):
    • В 2 раза большая скорость = в 2 раза большее расстояние реакции.
    • В 5 раз больше скорости = в 5 раз больше расстояние реакции.
  • Время вашей реакции.
    • Обычно 0,5–2 секунды.
    • Лучшее время реакции в пробках у людей в возрасте 45–54 лет.
    • У молодых людей в возрасте 18–24 лет и старше 60 лет одинаковое время реакции в условиях дорожного движения.У молодых людей более острые чувства, но у пожилых людей больше опыта.

Дальность реакции может быть уменьшена на

  • Предвидение опасностей.
  • Готовность.

Дальность реакции может быть увеличена на

Простой метод: вычислить расстояние реакции

Формула: Удалите последнюю цифру скорости, умножьте на время реакции, а затем на 3.

Пример расчета при скорости 50 км / ч и времени реакции 1 секунда:

50 км / ч ⇒ 5
5 * 1 * 3 = 15 метров расстояние реакции

Более точный метод: вычислить расстояние реакции

Формула: d = (s * r) / 3.6

d = расстояние реакции в метрах (рассчитывается).
с = скорость в км / ч.
r = время реакции в секундах.
3,6 = фиксированное значение для преобразования км / ч в м / с.

Пример расчета при скорости 50 км / ч и времени реакции 1 секунда:

(50 * 1) / 3,6 = 13,9 метра расстояние реакции

Тормозной путь

Тормозной путь — это расстояние, которое проходит автомобиль от момента начала торможения до остановки.

На тормозной путь влияет

  • Скорость автомобиля (квадратичное увеличение; «в 2 раза»):
    • Увеличение скорости в 2 раза = увеличение тормозного пути в 4 раза.
    • В 3 раза больше скорости = в 9 раз больше тормозной путь.
  • Дорога (уклон и условия).
  • Нагрузка.
  • Тормоза (состояние, тормозная техника и количество тормозных колес).

Рассчитать тормозной путь

Очень сложно добиться надежных расчетов тормозного пути, так как дорожные условия и сцепление шин с дорогой могут сильно различаться.Тормозной путь может быть, например, в 10 раз больше, когда на дороге есть лед.

Простой метод: расчет тормозного пути

Условия: Хорошие и сухие дорожные условия, хорошие шины и хорошие тормоза.

Формула: Удалите ноль из скорости, умножьте это число на себя, а затем умножьте на 0,4.

Цифра 0,4 взята из того факта, что тормозной путь с 10 км / ч в условиях сухой дороги составляет примерно 0.4 метра. Это было рассчитано с помощью исследователей, измеряющих тормозной путь. Таким образом, в упрощенной формуле мы основываем наши расчеты на тормозном пути при 10 км / ч и увеличиваем его квадратично с увеличением скорости.

Пример расчета при скорости 10 км / ч:

10 км / ч ⇒ 1
1 * 1 = 1
1 * 0,4 = 0,4 метра тормозной путь

Пример расчета при скорости 50 км / ч:

50 км / ч ⇒ 5
5 * 5 = 25
25 * 0.4 = 10 метров тормозной путь

Более точный метод: расчет тормозного пути

Состояние: Хорошая резина и хорошие тормоза.

Формула: d = s 2 / (250 * f)

d = тормозной путь в метрах (подлежит расчету).
с = скорость в км / ч.
250 = фиксированная цифра, которая используется всегда.
f = коэффициент трения, прибл. 0,8 по сухому асфальту и 0.1 на льду.

Пример расчета при скорости 50 км / ч по сухому асфальту:

50 2 / (250 * 0,8) = 12,5 метров тормозного пути

Тормозной путь

Тормозной путь = путь реакции + тормозной путь

Рассчитайте тормозной путь с помощью этих простых методов

Сейчас лето, дорога сухая. Вы едете со скоростью 90 км / ч на машине с хорошими шинами и тормозами. Вы внезапно замечаете опасность на дороге и резко тормозите.Какова длина тормозного пути, если время вашей реакции составляет 1 секунду?

Тормозной путь — это расстояние реакции + тормозной путь . Сначала рассчитываем расстояние реакции:

  • 90 км / ч ⇒ 9
  • .
  • 9 * 1 * 3 = 27 метров расстояние реакции

Затем рассчитываем тормозной путь:

  • 90 км / ч ⇒ 9
  • .
  • 9 * 9 = 81
  • 81 * 0,4 = 32 метра тормозной путь

Теперь оба расстояния объединены:

  • 27 + 32 = тормозной путь в метрах

Важное пояснение относительно расчетов

Различные методы дают разные ответы.Что мне использовать?
— Используйте то, что хотите. Различия настолько малы, что не повлияют на ваш теоретический тест, поскольку разница между альтернативами довольно велика.

Итак, если есть альтернативы 10, 20, 40, 60, не имеет значения, получите ли вы 10 метров одним методом и 12,5 метра другим — оба, очевидно, наиболее близки к 10, что, таким образом, является правильным ответом.

Последнее обновление 22.01.2021.

Понимание остановки и тормозного пути в физике — урок физики (видео)

Расстояние мышления (TD)

Давайте сначала начнем с расстояния мышления (TD), которое показано в уравнении 2.Можно считать, что скорость автомобиля постоянна в течение короткого промежутка времени, необходимого для реакции водителя, поэтому все, что нам нужно, — это скорость, умноженная на время реакции, чтобы получить расстояние мысли. Поскольку время реакции человека, желающего затормозить, обычно меньше секунды, это расстояние является наименьшим по отношению к тормозному пути.

Уравнение 2

Тормозной путь (BD)

Вывести уравнение для тормозного пути немного сложнее.Начнем с кинематического уравнения, показанного в уравнении 3.

Уравнение 3

Где:

  • vf = конечная скорость
  • vo = начальная скорость
  • a = ускорение
  • d = пройденное расстояние

Мы знаем, что конечная скорость равна нулю, потому что машина остановилась. Единственное неизвестное в этом уравнении — это ускорение a .Автомобиль замедляется (ускоряется в направлении, противоположном его движению), потому что на него действует неуравновешенная сила.

Тормоза создают трение колесам, замедляя их, но статическое трение ( f ) между колесами и дорогой в конечном итоге останавливает машину. Сопротивление воздуха и трение качения участвуют, но в меньшей степени. Вес автомобиля ( мг, ) и нормальная сила ( Н, ) являются вертикальными силами, и они равны.Схема свободного тела показана на Диаграмме 1.

Диаграмма 1.

Второй закон Ньютона используется для расчета ускорения автомобиля. Трение рассчитывается путем умножения коэффициента трения (μ) на нормальную силу ( Н, ).

f = μ N

Нормальная сила составляет мг , потому что она должна только противодействовать весу автомобиля. Последняя строка в уравнении 4 дает нам ускорение автомобиля.

Уравнение 4

Теперь мы можем подставить ускорение, которое мы только что определили, в уравнение 3, чтобы получить уравнение тормозного пути, BD. Давайте рассмотрим это более подробно.

Уравнение 5. g — ускорение свободного падения.

Последний шаг в нашем выводе уравнения тормозного пути (SD) — это прибавить мысленное расстояние (TD) к тормозному пути (BD), что показано в уравнении 6.

Уравнение 6

Давайте представим, что время реакции нашего водителя составляет 0,5 с, и мы знаем, что начальная скорость составляет 73 км / ч, что составляет 20,3 м / с. Коэффициент трения (μ) можно оценить как 0,8, что является средним значением для резиновых шин на сухом бетоне. Что теперь может определить наш минимальный тормозной путь?

Довольно удивительно, что за доли секунды наш мозг может сравнить значение тормозного пути с нашей оценкой того, как далеко мы от перекрестка, и принять решение, остановиться или проехать перекресток.Что ж, может быть, это не совсем то, что происходит, но с практикой вождения мы обучаемся точно оценивать расстояние, которое нам нужно, чтобы остановиться, в зависимости от нашей скорости.

Дальность мышления увеличивается со скоростью. Время нашей реакции может быть постоянным, но умножение его на все более и более высокие скорости увеличивает расстояние мышления с увеличением скорости. Тормозной путь увеличивается экспоненциально с увеличением скорости, потому что начальная скорость автомобиля возведена в квадрат в уравнении тормозного пути.Например, для остановки движения со скоростью 20 м / с требуется дополнительно 24 м по сравнению с 10 м / с. График 1 показывает тормозной путь в сравнении с начальными скоростями.

График 1

Резюме урока

Давайте сделаем несколько минут, чтобы повторить то, что мы узнали!

Каждый раз, когда кто-то водит машину, он должен в какой-то момент остановить ее. Это включает в себя принятие решения об остановке, во время которого автомобиль проезжает определенное расстояние, равное его мгновенной скорости, умноженной на время реакции водителя.Мы называем это расстояние расстоянием мышления (TD). Это кратчайшее расстояние в уравнении тормозного пути, потому что время реакции водителя очень мало.

Тормозной путь (BD) — это расстояние, необходимое для остановки после включения тормозов, а статическое трение между шинами и дорогой является доминирующей тормозящей силой, замедляющей автомобиль до полной остановки.

Сложение этих двух расстояний вместе дает нам тормозной путь (SD).

Самым большим фактором при оценке этого расстояния является скорость автомобиля, поскольку она возводится в квадрат в уравнениях тормозного пути и тормозного пути.

Как скорость влияет на тормозной путь

На тормозной путь может влиять множество различных факторов. Тормозной путь — это показатель того, как далеко ваш автомобиль проходит за время, необходимое для полной остановки после того, как вы нажмете на тормоз. Ваш тормозной путь будет короче (то есть лучше), если ваши тормоза и шины в хорошем состоянии.Это означает, что вы должны убедиться, что ваши шины имеют правильный уровень давления воздуха и достаточный протектор.

Независимо от того, насколько красив или ухожен ваш автомобиль, лучшим показателем вашего тормозного пути будет ваша скорость. Посмотрите, как скорость автомобиля изменяет тормозной путь.

Дистанция мышления

Дистанция обдумывания — это время, необходимое вам, чтобы решить нажать на тормоза, а затем фактически нажать на них. Когда вы видите на дороге потенциальную угрозу, знак или систему управления движением, вы не сломаетесь сразу.Вам может потребоваться секунда или две, чтобы поставить ногу на педаль тормоза. Время реакции может быть замедлено, если водитель сонный, болен, ослаблен или отвлекается. Каждый раз за рулем убедитесь, что вы трезвы и внимательны, потому что иногда дополнительная секунда может иметь решающее значение.

Тормозной путь

Тормозной путь — это время, за которое ваш автомобиль полностью остановится. после того, как вы нажали на тормоз. Когда вы вдвое увеличиваете скорость вашего автомобиля, ваш тормозной путь увеличивается в четыре раза.Как показано ниже, каждый раз, когда вы удваиваете скорость, вы умножаете свой тормозной путь на четыре. Эта информация будет важна для определения общего тормозного пути. Продолжай читать!

Общий тормозной путь

Уравнение для определения того, как далеко уедет ваша машина от момента, когда вы заметите опасность, до момента, когда вы полностью остановитесь:

[расстояние мыслей] + [тормозной путь] = [общий тормозной путь]

Чем быстрее вы едете, тем больше места вы преодолеете, отреагировав и начав тормозить.Имеет смысл, правда? Ознакомьтесь с приведенными ниже уравнениями общего тормозного пути для транспортных средств, движущихся с различной скоростью.

60 миль / ч: расстояние мышления 60 футов + тормозное расстояние 180 футов = общее расстояние 240 футов

40 миль / ч: расстояние мышления 40 футов + тормозное расстояние 80 футов = общее расстояние 120 футов

20 миль в час: расстояние мышления 20 футов + тормозное расстояние 20 футов = общее расстояние 40 футов

Хотите узнать больше о том, как быть безопасным водителем?

Расчет тормозного пути — Безопасность в транспортных средствах — Eduqas — GCSE Physics (Single Science) Revision — Eduqas

Важно уметь:

  • оценить, как тормозной путь транспортного средства изменяется в зависимости от скорости. сделано при остановке движущегося транспортного средства

На диаграмме показаны некоторые типичные тормозные пути для среднего автомобиля в нормальных условиях.

Гистограмма, показывающая мысленный и тормозной путь автомобиля на разных скоростях. Чем больше скорость, тем больше времени требуется на размышление и торможение.

Некоторые типичные тормозные пути

Движение со скоростью 20 миль / ч (32 км / ч):

  • мысленное расстояние = 6 м
  • тормозной путь = 6 м
  • общий тормозной путь = 12 м

Движение со скоростью 40 миль / ч (64 км / ч):

  • расстояние мышления = 12 м
  • тормозное расстояние = 24 м
  • общий тормозной путь = 36 м

Движение со скоростью 70 миль в час (112 км / ч):

  • расстояние мышления = 21 м
  • тормозной путь = 75 м
  • общий тормозной путь = 96 м

Важно отметить, что мысленное расстояние пропорционально стартовой скорости.Это означает, что оно увеличивается пропорционально увеличению скорости — то есть, если скорость удваивается, расстояние мышления также удваивается.

Однако тормозной путь увеличивается в четыре раза каждый раз, когда стартовая скорость удваивается.

Например, если автомобиль увеличивает скорость вдвое с 30 до 60 миль в час, дистанция мышления удвоится с 9 до 18 м, а тормозной путь увеличится в четыре раза с 14 до 56 м.

Тормозное усилие — большее

Тормозной путь увеличивается в четыре раза каждый раз, когда стартовая скорость удваивается.{2} \)

Итак, при фиксированной максимальной тормозной силе тормозной путь пропорционален квадрату скорости.

Пример расчета дистанции мышления

Автомобиль движется со скоростью 12 м / с. Водитель имеет время реакции 0,5 с и видит, что впереди на дорогу выбегает кошка. Какова дистанция мышления, когда водитель реагирует?

расстояние = скорость × время

\ [d = v \ times t \]

\ [d = 12 ~ м / с \ times 0,5 ~ s \]

\ [\ underline {мышление \ расстояние = 6 ~ m} \]

Пример расчета тормозного пути — выше

Автомобиль в предыдущем примере имеет общую массу 900 кг.{2}} {2,000} \]

\ [\ underline {braking \ distance = 32 \ m} \]

Пример расчета тормозного пути

Каков тормозной путь для автомобиля выше?

тормозной путь = расстояние мысли + тормозной путь

тормозной путь = 6 + 32

тормозной путь = 38 м

Вопрос

Рассчитайте тормозной путь для автомобиля и водителя в приведенном выше примере при движении на 24 м. / с.

Показать ответ

\ [мышление \ расстояние = 24 ~ м / с \ умножить на 0.{2}} {100} \]

\ [\ underline {braking \ force \ is ~ 87000 \ N} \]

Знаете ли вы, как рассчитать тормозной путь?

Что такое тормозной путь?

Тормозной путь — это расстояние, на котором ваш автомобиль останавливается после нажатия на педаль тормоза.

Расчет тормозного пути важен для предотвращения потенциальных аварий или опасных ситуаций.

Факторы, влияющие на расстояние

Существует множество переменных факторов, которые могут повлиять на то, как долго ваш автомобиль останавливается.Вот некоторые;

  • Скорость транспортного средства
  • Масса транспортного средства
  • Дорожные условия (скользкое, ледяное, снеговое, сухое, мокрое)
  • Условия торможения транспортного средства (старые или изношенные колодки и роторы)
  • Тормозная техника в транспортном средстве (например, как ABS)
  • Состояние шин (лысые шины)

Это приводит нас к фактической формуле для расчета тормозного пути.

Формула основана на скорости (скорости) транспортного средства и коэффициенте трения между колесами и дорогой.

Тормозной путь = Скорость² / 2 (коэффициент трения) (ускорение свободного падения)

Да, в любом случае, как только олень выбегает перед вами, вы думаете: «Итак, что такое 50 миль в час в квадрате… деленное на..?» splat.

Хорошо, это не работает. Если вы не гений физики или математики, это нереально.

Вот несколько более простая формула, опубликованная на сайте Майкла Никсона на сайте инструктора по вождению.

Предполагается, что сухая погода и хорошее покрытие.

20 миль / ч = x2 = 40 футов
30 миль / ч = x2,5 = 75 футов
40pmh = x3 = 120 футов
50 миль / ч = x3,5 = 175 футов
60 миль / ч = x4 = 240 футов
70 миль / ч = x4,5 = 315 футов

Удвойте (X2) , эти расстояния для мокрых дорог, и умноженные на десять ( X10) для снег / лед .

Рассчитайте тормозной путь EASY WAY.

Это не тормозной путь, но хорошее место для старта.

Согласно Smartmotorist.com, вы должны применить « правило трех секунд ». Это означает, что вы найдете неподвижный объект на обочине дороги, например, дорожный знак или отметку мили.

Когда впереди идущий автомобиль проезжает знак, сосчитайте до трех, и к этому времени вы уже проезжаете тот же дорожный знак / объект, который выбрали.

При скорости 65 миль в час вы можете путешествовать почти 100 футов в секунду.

Правило трех секунд должно дать вам расстояние около 288 футов между движущимся впереди автомобилем.

Это основано на хороших сухих условиях. Увеличьте количество секунд для мокрой дороги и ненастной погоды.

Дважды для мокрой дороги (6 секунд) и десять раз для снега (30 секунд).

Соблюдайте безопасную дистанцию ​​следования ». Соблюдайте безопасную дистанцию ​​следования (правило трех секунд) . Умный автомобилист, н.д. Интернет. 21 апреля 2017 г. .

Никсон, Майкл.«Простой способ рассчитать тормозные пути». Простой способ выработать тормозные пути | Майк Никсон . N.p., n.d. Интернет. 21 апр.2017 г.

Калькулятор (общего) остановочного пути / тормозного пути

Этот онлайн-калькулятор тормозного пути разработан для широкого спектра применений и может рассчитывать два из следующих пяти размеров — в зависимости от состояние дороги или трассы: тормозной путь и общий тормозной путь, время (торможение), начальная скорость, конечная скорость и ускорение / замедление.Кроме того, мышление расстояние тоже рассчитывается.

Этот калькулятор , , , не использует знакомые формулы первого раза. Расчет производится по точным формулам.

По умолчанию Тормозной путь и общий тормозной путь рассчитываются для следующих условий: начальная скорость 100 км / ч, проезжая часть должна быть сухой, чистой, плоский, прямой и герметичный. Время реакции — одна секунда.

>> Формулы для скорости, ускорения, времени и расстояния

В условии «» вы найдете подходящие значения для автомобильных и железнодорожных транспортных средств.

* Измените состояние железной дороги или дороги в первом калькуляторе; предположение: все колеса заторможены.

** Возможны и отрицательные значения!

*** Значения двух сокращений l и h можно найти здесь: Общая информация.

Автомобиль едет по сухой ровной дороге через деревню. Его скорость составляет 50 км / ч. Вдруг на улицу прыгает ребенок.

  1. На каком расстоянии от ребенка должна находиться машина, чтобы она могла вовремя остановиться? Предполагается, что время реакции составляет одну секунду.
  2. Сейчас дорога мокрая. Какая скорость у машины во время аварии?
  3. Водитель нелегально проезжает по поселку со скоростью 60 км / ч. Тормозной путь должен быть таким же, как в а).Какая скорость у машины, когда она наезжает? ребенок?
  4. Начальная скорость автомобиля — 50 км / ч, время реакции — две секунды. Какой тормозной путь автомобиль есть?

Ответ а)

В поле «начальная скорость» вы должны ввести значение 50 вместо 100. Затем нажмите «Рассчитать» или клавишу Enter. Чтобы успеть вовремя, автомобиль должен находиться на расстоянии 24,812 м. от ребенка. Конечно, для правильного ответа необходимо соблюдать тормозной путь.

Ответ б)

Если вы хотите узнать скорость столкновения, необходимо заполнить калькулятор тормозного пути, как показано на скриншоте ниже:

После ввода значений не нажимайте сразу кнопку «Рассчитать». Сначала измените условие на «Мокрая дорога» (щелкните маленькую стрелку). Результат 33 км / ч.

Ответ c)

Для этого расчета вы должны ввести значение 50 в поле «начальная скорость».Пусть все остальные значения как есть — затем выберите «Сухая проезжая часть». В этих условиях машина по-прежнему развивает скорость около 42 км / ч!

Ответ г)

Сначала сбросьте калькулятор. В качестве «Время реакции» выберите 2 и замените «Начальную скорость» на значение 50. Общий тормозной путь составляет 38,701 м, тормозной путь — 10,923 м. В разница между двумя числами дает расстояние реакции: 27,778 м.

В этом случае расстояние реакции уже больше, чем полный тормозной путь из пункта а).Так что машина на полной скорости врезается в препятствие!

Общий тормозной путь — это сумма тормозного пути и дистанции мышления. Общее время торможения складывается из времени реакции и времени торможения.

Добавить комментарий

Ваш адрес email не будет опубликован. Обязательные поля помечены *